What's an intuitive way to think about the determinant?












584












$begingroup$


In my linear algebra class, we just talked about determinants. So far I’ve been understanding the material okay, but now I’m very confused. I get that when the determinant is zero, the matrix doesn’t have an inverse. I can find the determinant of a $2times 2$ matrix by the formula. Our teacher showed us how to compute the determinant of an $N times N$ matrix by breaking it up into the determinants of smaller matrices, and apparently there is a way by summing over a bunch of permutations. But the notation is really hard for me and I don’t really know what’s going on with them anymore. Can someone help me figure out what a determinant is, intuitively, and how all those definitions of it are related?










share|cite|improve this question











$endgroup$








  • 77




    $begingroup$
    I just wanted this question to be in the archives, because it's a perennial one that admits a better response than is in most textbooks.
    $endgroup$
    – Jamie Banks
    Jul 25 '10 at 2:26






  • 6




    $begingroup$
    Hehe, you're going against your own suggestion of asking questions that you actually want answered!! I'm teasing though, I understand your motivation. Can we set a precedent of making seeded questions CW? I kinda like that idea, I will propose it on Meta. I am rambling.
    $endgroup$
    – BBischof
    Jul 25 '10 at 2:34






  • 6




    $begingroup$
    In case somebody was curious about the trace form of this question, it is more difficult, and is the subject of one of my MO questions :D mathoverflow.net/questions/13526/…
    $endgroup$
    – BBischof
    Jul 25 '10 at 2:36






  • 3




    $begingroup$
    I'm confused - did the Katie Banks answer her own question?
    $endgroup$
    – user1729
    Nov 14 '11 at 15:24






  • 4




    $begingroup$
    Those reading this forum in '16 may be interested in this video The Determinant which is part of a set of videos that give some very nice insight into intuitive understanding of linear algebra (the essence of it rather)
    $endgroup$
    – Hugh Entwistle
    Oct 11 '16 at 6:43


















584












$begingroup$


In my linear algebra class, we just talked about determinants. So far I’ve been understanding the material okay, but now I’m very confused. I get that when the determinant is zero, the matrix doesn’t have an inverse. I can find the determinant of a $2times 2$ matrix by the formula. Our teacher showed us how to compute the determinant of an $N times N$ matrix by breaking it up into the determinants of smaller matrices, and apparently there is a way by summing over a bunch of permutations. But the notation is really hard for me and I don’t really know what’s going on with them anymore. Can someone help me figure out what a determinant is, intuitively, and how all those definitions of it are related?










share|cite|improve this question











$endgroup$








  • 77




    $begingroup$
    I just wanted this question to be in the archives, because it's a perennial one that admits a better response than is in most textbooks.
    $endgroup$
    – Jamie Banks
    Jul 25 '10 at 2:26






  • 6




    $begingroup$
    Hehe, you're going against your own suggestion of asking questions that you actually want answered!! I'm teasing though, I understand your motivation. Can we set a precedent of making seeded questions CW? I kinda like that idea, I will propose it on Meta. I am rambling.
    $endgroup$
    – BBischof
    Jul 25 '10 at 2:34






  • 6




    $begingroup$
    In case somebody was curious about the trace form of this question, it is more difficult, and is the subject of one of my MO questions :D mathoverflow.net/questions/13526/…
    $endgroup$
    – BBischof
    Jul 25 '10 at 2:36






  • 3




    $begingroup$
    I'm confused - did the Katie Banks answer her own question?
    $endgroup$
    – user1729
    Nov 14 '11 at 15:24






  • 4




    $begingroup$
    Those reading this forum in '16 may be interested in this video The Determinant which is part of a set of videos that give some very nice insight into intuitive understanding of linear algebra (the essence of it rather)
    $endgroup$
    – Hugh Entwistle
    Oct 11 '16 at 6:43
















584












584








584


414



$begingroup$


In my linear algebra class, we just talked about determinants. So far I’ve been understanding the material okay, but now I’m very confused. I get that when the determinant is zero, the matrix doesn’t have an inverse. I can find the determinant of a $2times 2$ matrix by the formula. Our teacher showed us how to compute the determinant of an $N times N$ matrix by breaking it up into the determinants of smaller matrices, and apparently there is a way by summing over a bunch of permutations. But the notation is really hard for me and I don’t really know what’s going on with them anymore. Can someone help me figure out what a determinant is, intuitively, and how all those definitions of it are related?










share|cite|improve this question











$endgroup$




In my linear algebra class, we just talked about determinants. So far I’ve been understanding the material okay, but now I’m very confused. I get that when the determinant is zero, the matrix doesn’t have an inverse. I can find the determinant of a $2times 2$ matrix by the formula. Our teacher showed us how to compute the determinant of an $N times N$ matrix by breaking it up into the determinants of smaller matrices, and apparently there is a way by summing over a bunch of permutations. But the notation is really hard for me and I don’t really know what’s going on with them anymore. Can someone help me figure out what a determinant is, intuitively, and how all those definitions of it are related?







linear-algebra matrices determinant intuition






share|cite|improve this question















share|cite|improve this question













share|cite|improve this question




share|cite|improve this question








edited Oct 29 '17 at 19:08









Rodrigo de Azevedo

13.1k41960




13.1k41960










asked Jul 25 '10 at 2:25









Jamie BanksJamie Banks

6,37173036




6,37173036








  • 77




    $begingroup$
    I just wanted this question to be in the archives, because it's a perennial one that admits a better response than is in most textbooks.
    $endgroup$
    – Jamie Banks
    Jul 25 '10 at 2:26






  • 6




    $begingroup$
    Hehe, you're going against your own suggestion of asking questions that you actually want answered!! I'm teasing though, I understand your motivation. Can we set a precedent of making seeded questions CW? I kinda like that idea, I will propose it on Meta. I am rambling.
    $endgroup$
    – BBischof
    Jul 25 '10 at 2:34






  • 6




    $begingroup$
    In case somebody was curious about the trace form of this question, it is more difficult, and is the subject of one of my MO questions :D mathoverflow.net/questions/13526/…
    $endgroup$
    – BBischof
    Jul 25 '10 at 2:36






  • 3




    $begingroup$
    I'm confused - did the Katie Banks answer her own question?
    $endgroup$
    – user1729
    Nov 14 '11 at 15:24






  • 4




    $begingroup$
    Those reading this forum in '16 may be interested in this video The Determinant which is part of a set of videos that give some very nice insight into intuitive understanding of linear algebra (the essence of it rather)
    $endgroup$
    – Hugh Entwistle
    Oct 11 '16 at 6:43
















  • 77




    $begingroup$
    I just wanted this question to be in the archives, because it's a perennial one that admits a better response than is in most textbooks.
    $endgroup$
    – Jamie Banks
    Jul 25 '10 at 2:26






  • 6




    $begingroup$
    Hehe, you're going against your own suggestion of asking questions that you actually want answered!! I'm teasing though, I understand your motivation. Can we set a precedent of making seeded questions CW? I kinda like that idea, I will propose it on Meta. I am rambling.
    $endgroup$
    – BBischof
    Jul 25 '10 at 2:34






  • 6




    $begingroup$
    In case somebody was curious about the trace form of this question, it is more difficult, and is the subject of one of my MO questions :D mathoverflow.net/questions/13526/…
    $endgroup$
    – BBischof
    Jul 25 '10 at 2:36






  • 3




    $begingroup$
    I'm confused - did the Katie Banks answer her own question?
    $endgroup$
    – user1729
    Nov 14 '11 at 15:24






  • 4




    $begingroup$
    Those reading this forum in '16 may be interested in this video The Determinant which is part of a set of videos that give some very nice insight into intuitive understanding of linear algebra (the essence of it rather)
    $endgroup$
    – Hugh Entwistle
    Oct 11 '16 at 6:43










77




77




$begingroup$
I just wanted this question to be in the archives, because it's a perennial one that admits a better response than is in most textbooks.
$endgroup$
– Jamie Banks
Jul 25 '10 at 2:26




$begingroup$
I just wanted this question to be in the archives, because it's a perennial one that admits a better response than is in most textbooks.
$endgroup$
– Jamie Banks
Jul 25 '10 at 2:26




6




6




$begingroup$
Hehe, you're going against your own suggestion of asking questions that you actually want answered!! I'm teasing though, I understand your motivation. Can we set a precedent of making seeded questions CW? I kinda like that idea, I will propose it on Meta. I am rambling.
$endgroup$
– BBischof
Jul 25 '10 at 2:34




$begingroup$
Hehe, you're going against your own suggestion of asking questions that you actually want answered!! I'm teasing though, I understand your motivation. Can we set a precedent of making seeded questions CW? I kinda like that idea, I will propose it on Meta. I am rambling.
$endgroup$
– BBischof
Jul 25 '10 at 2:34




6




6




$begingroup$
In case somebody was curious about the trace form of this question, it is more difficult, and is the subject of one of my MO questions :D mathoverflow.net/questions/13526/…
$endgroup$
– BBischof
Jul 25 '10 at 2:36




$begingroup$
In case somebody was curious about the trace form of this question, it is more difficult, and is the subject of one of my MO questions :D mathoverflow.net/questions/13526/…
$endgroup$
– BBischof
Jul 25 '10 at 2:36




3




3




$begingroup$
I'm confused - did the Katie Banks answer her own question?
$endgroup$
– user1729
Nov 14 '11 at 15:24




$begingroup$
I'm confused - did the Katie Banks answer her own question?
$endgroup$
– user1729
Nov 14 '11 at 15:24




4




4




$begingroup$
Those reading this forum in '16 may be interested in this video The Determinant which is part of a set of videos that give some very nice insight into intuitive understanding of linear algebra (the essence of it rather)
$endgroup$
– Hugh Entwistle
Oct 11 '16 at 6:43






$begingroup$
Those reading this forum in '16 may be interested in this video The Determinant which is part of a set of videos that give some very nice insight into intuitive understanding of linear algebra (the essence of it rather)
$endgroup$
– Hugh Entwistle
Oct 11 '16 at 6:43












13 Answers
13






active

oldest

votes


















344












$begingroup$

Your trouble with determinants is pretty common. They’re a hard thing to teach well, too, for two main reasons that I can see: the formulas you learn for computing them are messy and complicated, and there’s no “natural” way to interpret the value of the determinant, the way it’s easy to interpret the derivatives you do in calculus at first as the slope of the tangent line. It’s hard to believe things like the invertibility condition you’ve stated when it’s not even clear what the numbers mean and where they come from.



Rather than show that the many usual definitions are all the same by comparing them to each other, I’m going to state some general properties of the determinant that I claim are enough to specify uniquely what number you should get when you put in a given matrix. Then it’s not too bad to check that all of the definitions for determinant that you’ve seen satisfy those properties I’ll state.



The first thing to think about if you want an “abstract” definition of the determinant to unify all those others is that it’s not an array of numbers with bars on the side. What we’re really looking for is a function that takes N vectors (the N columns of the matrix) and returns a number. Let’s assume we’re working with real numbers for now.



Remember how those operations you mentioned change the value of the determinant?




  1. Switching two rows or columns changes the sign.


  2. Multiplying one row by a constant multiplies the whole determinant by that constant.


  3. The general fact that number two draws from: the determinant is linear in each row. That is, if you think of it as a function $det: mathbb{R}^{n^2} rightarrow mathbb{R}$, then $$ det(a vec v_1 +b vec w_1 , vec v_2 ,ldots,vec v_n ) = a det(vec v_1,vec v_2,ldots,vec v_n) + b det(vec w_1, vec v_2, ldots,vec v_n),$$ and the corresponding condition in each other slot.


  4. The determinant of the identity matrix $I$ is $1$.



I claim that these facts are enough to define a unique function that takes in N vectors (each of length N) and returns a real number, the determinant of the matrix given by those vectors. I won’t prove that, but I’ll show you how it helps with some other interpretations of the determinant.



In particular, there’s a nice geometric way to think of a determinant. Consider the unit cube in N dimensional space: the set of vectors of length N with coordinates 0 or 1 in each spot. The determinant of the linear transformation (matrix) T is the signed volume of the region gotten by applying T to the unit cube. (Don’t worry too much if you don’t know what the “signed” part means, for now).



How does that follow from our abstract definition?



Well, if you apply the identity to the unit cube, you get back the unit cube. And the volume of the unit cube is 1.



If you stretch the cube by a constant factor in one direction only, the new volume is that constant. And if you stack two blocks together aligned on the same direction, their combined volume is the sum of their volumes: this all shows that the signed volume we have is linear in each coordinate when considered as a function of the input vectors.



Finally, when you switch two of the vectors that define the unit cube, you flip the orientation. (Again, this is something to come back to later if you don’t know what that means).



So there are ways to think about the determinant that aren’t symbol-pushing. If you’ve studied multivariable calculus, you could think about, with this geometric definition of determinant, why determinants (the Jacobian) pop up when we change coordinates doing integration. Hint: a derivative is a linear approximations of the associated function, and consider a “differential volume element” in your starting coordinate system.



It’s not too much work to check that the area of the parallelogram formed by vectors $(a,b)$ and $(c,d)$ is $Big|{}^{a;b}_{c;d}Big|$
either: you might try that to get a sense for things.






share|cite|improve this answer











$endgroup$









  • 22




    $begingroup$
    Very nice explanation. I think you should clarify what you mean by "the determinate is linear in each row" though
    $endgroup$
    – Casebash
    Jul 25 '10 at 3:07






  • 2




    $begingroup$
    Great answer. We were taught the determinant as the generalized volume function in our algebra class.
    $endgroup$
    – Neil G
    Aug 28 '10 at 9:26






  • 15




    $begingroup$
    Just out of curiosity, who are you talking to with the first sentence? Didn't you ask the question?!? Either way, I point students to this Q (&A) all the time for determinant help.
    $endgroup$
    – The Chaz 2.0
    Apr 18 '12 at 2:18






  • 9




    $begingroup$
    @TheChaz this question was asked near the beginnings of Math.SE, when there was a need to populate the site with questions before it was opened up to 'the public'. In any case, answering your own questions is explicitly encouraged nowadays.
    $endgroup$
    – Chris Taylor
    May 24 '12 at 7:51






  • 12




    $begingroup$
    To see that the geometric interpretation (volume of the image of cube) satisfies the multilinearity property it is not enough to stack two "aligned" blocks, this is again the multiplication of a column by a scalar. You should deal with two "not aligned" blocks produced by changing a single column vector, and see that the sum of their volumes is the volume of the block obtained putting the sum of the two vectors. This is not so easy to catch...
    $endgroup$
    – Emanuele Paolini
    Feb 20 '13 at 9:19



















194












$begingroup$

You could think of a determinant as a volume. Think of the columns of the matrix as vectors at the origin forming the edges of a skewed box. The determinant gives the volume of that box. For example, in 2 dimensions, the columns of the matrix are the edges of a rhombus.



You can derive the algebraic properties from this geometrical interpretation. For example, if two of the columns are linearly dependent, your box is missing a dimension and so it's been flattened to have zero volume.






share|cite|improve this answer











$endgroup$









  • 51




    $begingroup$
    If I may, I would add to this answer (which I think is a very good one) in two minor aspects. First, a determinant also has a sign, so we want the concept of oriented volume. (This is somewhat tricky, but definitely important, so you might as well have it in mind when you're learning about "right hand rules" and such.) Second, I think better than a volume is thinking of the determinant as the multiplicative change in volume of a parallelopiped under the linear transformation. (Of course you can always take the first one to be the unit n-cube and say that you are just dividing by one.)
    $endgroup$
    – Pete L. Clark
    Jul 28 '10 at 20:08






  • 8




    $begingroup$
    +1: I like this answer because there is a direct link to some application in physics: In special relativity we are talking of the conservation of space-time-volume, which means that the determinant of the transformation matrix is const. 1
    $endgroup$
    – vonjd
    Jan 16 '11 at 10:22



















97












$begingroup$

In addition to the answers, above, the determinant is a function from the set of square matrices into the real numbers that preserves the operation of multiplication:
begin{equation}det(AB) = det(A)det(B) end{equation}
and so it carries $some$ information about square matrices into the much more familiar set of real numbers.



Some examples:



The determinant function maps the identity matrix $I$ to the identity element of the real numbers ($det(I) = 1$.)



Which real number does not have a multiplicative inverse? The number 0. So which square matrices do not have multiplicative inverses? Those which are mapped to 0 by the determinant function.



What is the determinant of the inverse of a matrix? The inverse of the determinant, of course. (Etc.)



This "operation preserving" property of the determinant explains some of the value of the determinant function and provides a certain level of "intuition" for me in working with matrices.






share|cite|improve this answer











$endgroup$









  • 7




    $begingroup$
    +1 for including the questions. Many of them. Good ones. Especially the "So which square matrices do not have multiplicative inverses?" pair. And for featuring a nice doggy in your portrait!
    $endgroup$
    – n611x007
    Jan 30 '13 at 17:40





















37












$begingroup$

Here is a recording of my lecture on the geometric definition of determinants:



Geometric definition of determinants



It has elements from the answers by Jamie Banks and John Cook, and goes into details in a leisurely manner.






share|cite|improve this answer











$endgroup$









  • 1




    $begingroup$
    +1 for a detailed explaination video. You should put the disclosure that you are the lecturer. At first I thought you are a student in the class. By the way, I would like to see you make a full lectures list about linear algebra. It would be very nice.
    $endgroup$
    – Anh Tuan
    Jul 11 '17 at 9:14










  • $begingroup$
    This should be higher IMO. I think to have an intuition (geometrical understanding) of the determinant, you need a geometrical understanding of matrices first.
    $endgroup$
    – gwg
    Nov 1 '18 at 14:03



















33












$begingroup$

I too find the way determinants are treated in exterior algebra most intuitive. The definition is given on page 46 of Landsberg's "Tensors: Geometry and Applications". Two examples below will tell you everything you need to know.



Say, you are give a matrix
$$A=begin{pmatrix}a&b\c&dend{pmatrix}$$
and asked to compute its determinant. You can think of the matrix as a linear operator $f:mathbb R^2tomathbb R^2$ defined by



$$begin{pmatrix}x\yend{pmatrix}mapstobegin{pmatrix}a&b\c&dend{pmatrix}begin{pmatrix}x\yend{pmatrix}.$$



If you define the standard basis vector by $e_1=begin{pmatrix}1\0end{pmatrix}$ and $e_2=begin{pmatrix}0\1end{pmatrix}$, you can then define $f$ by the values it assumes on the basis vectors: $f(e_1)=ae_1+ce_2$ and $f(e_2)=be_1+de_2$.



The linear operator $f$ is extended to bivectors by
$$f(e_1wedge e_2)=f(e_1)wedge f(e_2).$$



Then you can write



$$f(e_1wedge e_2)=(ae_1+ce_2)wedge(be_1+de_2)=(ad-bc)e_1wedge e_2,$$



where I used distributivity and anticommutativity of the wedge product (the latter implies $awedge a=0$ for any vector $a$). So, we get the determinant as a scalar factor in the above equation, that is



$$f(e_1wedge e_2)=det(A),e_1wedge e_2.$$



The same procedure works for 3-by-3 matrices, you just need to use a trivector. Say, you are given $$B=begin{pmatrix}a_{11}&a_{12}&a_{13}\a_{21}&a_{22}&a_{23}\a_{31}&a_{32}&a_{33}end{pmatrix}.$$



It defines a linear operator $g:mathbb R^3to mathbb R^3$



$$begin{pmatrix}x\y\zend{pmatrix}mapsto begin{pmatrix}a_{11}&a_{12}&a_{13}\a_{21}&a_{22}&a_{23}\a_{31}&a_{32}&a_{33}end{pmatrix} begin{pmatrix}x\y\zend{pmatrix},$$



for which we have



$$g(e_1)=a_{11}e_1+a_{21}e_2+a_{31}e_3,quad g(e_2)=a_{12}e_1+a_{22}e_2+a_{32}e_3,quad g(e_3)=a_{13}e_1+a_{23}e_2+a_{33}e_3$$



on the standard basis $e_1=begin{pmatrix}1\0\0end{pmatrix}$, $e_2=begin{pmatrix}0\1\0end{pmatrix}$, $e_3=begin{pmatrix}0\0\1end{pmatrix}$. The operator $g$ is extended to trivectors by
$$g(e_1wedge e_2wedge e_3)=g(e_1)wedge g(e_2)wedge g(e_3),$$



which gives



$$g(e_1wedge e_2wedge e_3)=(a_{11}e_1+a_{21}e_2+a_{31}e_3)wedge(a_{12}e_1+a_{22}e_2+a_{32}e_3)wedge(a_{13}e_1+a_{23}e_2+a_{33}e_3).$$



If you then follows the rules of $wedge$ such as distributivity, anticommutativity, and associativity, you get
$$g(e_1wedge e_2wedge e_3)=det(B), e_1wedge e_2wedge e_3.$$



It works in exactly the same way in higher dimensions.






share|cite|improve this answer











$endgroup$









  • 2




    $begingroup$
    Beautiful answer. As someone who doesn't know any exterior algebra, this also serves as a motivation for the wedge product.
    $endgroup$
    – 6005
    Nov 15 '16 at 1:37





















27












$begingroup$

The top exterior power of an $n$-dimensional vector space $V$ is one-dimensional. Its elements are sometimes called pseudoscalars, and they represent oriented $n$-dimensional volume elements.



A linear operator $f$ on $V$ can be extended to a linear map on the exterior algebra according to the rules $f(alpha) = alpha$ for $alpha$ a scalar and $f(A wedge B) = f(A) wedge f(B), f(A + B) = f(A) + f(B)$ for $A$ and $B$ blades of arbitrary grade. Trivia: some authors call this extension an outermorphism. The extended map will be grade-preserving; that is, if $A$ is a homogeneous element of the exterior algebra of grade $m$, then $f(A)$ will also have grade $m$. (This can be verified from the properties of the extended map I just listed.)



All this implies that a linear map on the exterior algebra of $V$ once restricted to the top exterior power reduces to multiplication by a constant: the determinant of the original linear transformation. Since pseudoscalars represent oriented volume elements, this means that the determinant is precisely the factor by which the map scales oriented volumes.






share|cite|improve this answer











$endgroup$









  • 9




    $begingroup$
    It's worth mentioning here that, as abstract as the construction by the top exterior power is, it's the cleanest way to derive the permutation formula.
    $endgroup$
    – Qiaochu Yuan
    Jul 31 '10 at 8:14










  • $begingroup$
    I give a couple of examples of how this works in my answer.
    $endgroup$
    – Andrey Sokolov
    Nov 11 '13 at 4:26



















26












$begingroup$

For the record I'll try to give a reply to this old question, since I think some elements can be added to what has been already said.



Even though they are basically just (complicated) expressions, determinants can be mysterious when first encountered. Questions that arise naturally are: (1) how are they defined in general?, (2) what are their important properties?, (3) why do they exist?, (4) why should we care?, and (5) why does their expression get so huge for large matrices?



Since $2times2$ and $3times3$ determinants are easily defined explicitly, question (1) can wait. While (2) has many answers, the most important ones are, to me: determinants detect (by becoming 0) the linear dependence of $n$ vectors in dimension $n$, and they are an expression in the coordinates of those vectors (rather than for instance an algorithm). If you have a family of vectors that depend (or at least one of them depends) on a parameter, and you need to know for which parameter values they are linearly dependent, than trying to use for instance Gaussian elimination to detect linear dependence can run into trouble: one might need assumptions on the parameter to assure some coefficient is nonzero, and even then dividing by it gives very messy expressions. Provided the number of vectors equals the dimension $n$ of the space, taking a determinant will however immediately transform the question into an equation for the parameter (which one may or may not be capable of solving, but that is another matter). This is exactly how one obtains an equation in eigenvalue problems, in case you've seen those. This provides a first answer to (4). (But there is a lot more you can do with determinants once you get used to them.)



As for question (3), the mystery of why determinants exist in the first place can be reduced by considering the situation where one has $n-1$ given linearly independent vectors, and asks when a final unknown vector $vec x$ will remain independent from them, in terms of its coordinates. The answer is that it usually will, in fact always unless $vec x$ happens to be in the linear span $S$ of those $n-1$ vectors, which is a subspace of dimension $n-1$. For instance, if $n=2$ (with one vector $vec v$ given) the answer is "unless $vec x$ is a scalar multiple of $vec v$". Now if one imagines a fixed (nonzero) linear combination of the coordinates of $vec x$ (the technical term is a linear form on the space), then it will become $0$ precisely when $vec x$ is in some subspace of dimension $n-1$. With some luck, this can be arranged to be precisely the linear span $S$. (In fact no luck is involved: if one extends the $n-1$ vectors by one more vector to a basis, then expressing $vec x$ in that basis and taking its final coordinate will define such a linear form; however you can ignore this argument unless you are particularly suspicious.) Now the crucial observation is that not only does such a linear combination exist, its coefficients can be taken to be expressions in the coordinates of our $n-1$ vectors. For instance in the case $n=2$ if one puts $vec v={achoose b}$ and $vec x={x_1choose x_2}$, then the linear combination $-bx_1+ax_2$ does the job (it becomes 0 precisely when $vec x$ is a scalar multiple of $vec v$), and $-b$ and $a$ are clearly expressions in the coordinates of $vec v$. In fact they are linear expressions. For $n=3$ with two given vectors, the expressions for the coefficients of the linear combination are more complicated, but they can still be explicitly written down (each coefficient is the difference of two products of coordinates, one form each vector). These expressions are linear in each of the vectors, if the other one is fixed.



Thus one arrives at the notion of a multilinear expression (or form). The determinant is in fact a multilinear form: an expression that depends on $n$ vectors, and is linear in each of them taken individually (fixing the other vectors to arbitrary values). This means it is a sum of terms, each of which is the product of a coefficient, and of one coordinate each of all the $n$ vectors. But even ignoring the coefficients, there are many such terms possible: a whopping $n^n$ of them!



However, we want an expression that becomes $0$ when the vectors are linearly dependent. Now the magic (sort of) is that even the seemingly much weaker requirement that the expression becomes $0$ when two successive vectors among the $n$ are equal will assure this, and it will moreover almost force the form of our expression upon us. Multilinear forms that satisfy this requirement are called alternating. I'll skip the (easy) arguments, but an alternating form cannot involve terms that take the same coordinate of any two different vectors, and they must change sign whenever one interchanges the role of two vectors (in particular they cannot be symmetric with respect to the vectors, even though the notion of linear dependence is symmetric; note that already $-bx_1+ax_2$ is not symmetric with respect to interchange of $(a,b)$ and $(x_1,x_2)$). Thus any one term must involve each of the $n$ coordinates once, but not necessarily in order: it applies a permutation of the coordinates $1,2,ldots,n$ to the successive vectors. Moreover, if a term involves one such permutation, then any term obtained by interchanging two positions in the permutation must also occur, with an opposite coefficient. But any two permutations can be transformed into one another by repeatedly interchanging two positions; so if there are any terms at all, then there must be terms for all $n!$ permutations, and their coefficients are all equal or opposite. This explains question (5), why the determinant is such a huge expression when $n$ is large.



Finally the fact that determinants exist turns out to be directly related to the fact that signs can be associated to all permutations in such a way that interchanging entries always changes the sign, which is part of the answer to question (3).
As for question (1), we can now say that the determinant is uniquely determined by being an $n$-linear alternating expression in the entries of $n$ column vectors, which contains a term consisting of the product of their coordinates $1,2,ldots,n$ in that order (the diagonal term) with coefficient $+1$. The explicit expression is a sum over all $n!$ permutations, the corresponding term being obtained by applying those coordinates in permuted order, and with the sign of the permutation as coefficient. A lot more can be said about question (2), but I'll stop here.






share|cite|improve this answer











$endgroup$





















    12












    $begingroup$

    If you have a matrix




    • $H$
      then you can calculate the correlationmatrix with

    • $G = H times H^H$
      (H^H denotes the complex conjugated and transposed version of $H$).


    If you do a eigenvalue decomposition of $G$ you get eigenvalues $lambda$ and eigenvectors $v$, that in combination $lambdatimes v$ describes the same space.



    Now there is the following equation, saying:




    • Determinant($H*H^H$) = Product of all eigenvalues $lambda$


    I.e., if you have a $3times3$ matrix $H$ then $G$ is $3times3$ too giving us three eigenvalues.
    The product of these eigenvalues give as the volume of a cuboid.
    With every extra dimension/eigenvalue the cuboid gets an extra dimension.






    share|cite|improve this answer











    $endgroup$





















      12












      $begingroup$

      There are excellent answers here that are very detailed.



      Here I provide a simpler answer, also discussed in wikipedia. Think of the determinant as the area (in 2D; in 3D it would be the volume, etc.) of the parallelogram made by the vectors:



      Parallelogram from vectors



      Keep in mind that the area of a parallelogram is the base $times$ height. Doing some tricks with the dot product, this yields the determinant:



      $$
      begin{vmatrix}
      a & b \
      c & d
      end{vmatrix}
      = ad - bc = Area_{parallelogram}
      $$



      You can place the unit vectors for each dimension to test the identity matrix by seeing that:
      $$
      begin{vmatrix}
      1 & 0 \
      0 & 1
      end{vmatrix}
      = ad - bc = 1 times 1 - 0 times 0 = 1 $$



      This is a volume in more than a 2 by 2 matrix and will be equal to 1 in all cases as the off diagonal elements remove any effect from the only value contributing to the volume as the diagonal product of 1s. It is understood in some contexts that the coordinate system is unmodified.






      share|cite|improve this answer











      $endgroup$





















        6












        $begingroup$

        (I considered making this a comment, but I thought it might deserve more attention than a comment would receive. Upvotes and downvotes will tell if I am right or wrong).



        Complement about the sign of the determinant



        I loved the accepted answer by Jamie, but I was frustrated that it did not give more explanation about the sign of the determinant and the notion of "rotation" or "orientation" of a vector. The answer from Marc Van Leeuwen comments more on this, but maybe not enough for everyone -- at least for me -- to understand what it means for a matrix to change the orientation of the space it transforms. So I googled the issue and ended up on the following explanation which I find excellent and accessible:



        http://mathinsight.org/determinant_linear_transformation#lintrans3D






        share|cite|improve this answer











        $endgroup$





















          4












          $begingroup$

          Think about a scalar equation,
          $$ax = b$$
          where we want to solve for $x$. We know we can always solve the equation if $aneq 0$, however, if $a=0$ then the answer is "it depends". If $bneq 0$, then we cannot solve it, however, if $b=0$ then there are many solutions (i.e. $x in mathbb{R}$). The key point is that the ability to solve the equation unambiguously depends on whether $a=0$.



          When we consider the similar equation for matrices



          $$mathbf{Ax} = mathbf{b}$$



          the question as to whether we can solve it is not so easily settled by whether $mathbf{A}=mathbf{0}$ because $mathbf{A}$ could consist of all non-zero elements and still not be solvable for $mathbf{b}neqmathbf{0}$. In fact, for two different vectors $mathbf{y}_1 neq mathbf{0}$ and $mathbf{y}_2neq mathbf{0}$ we could very well have that



          $$mathbf{Ay}_1 neq mathbf{0}$$
          and
          $$mathbf{Ay}_2 = mathbf{0}.$$



          If we think of $mathbf{y}$ as a vector, then there are some directions in which $mathbf{A}$ behaves like non-zero (this is called the row space) and other directions where $mathbf{A}$ behaves like zero (this is called the null space). The bottom line is that if $mathbf{A}$ behaves like zero in some directions, then the answer to the question "is $mathbf{Ax} = mathbf{b}$ generally solvable for any $mathbf{b}$?" is "it depends on $mathbf{b}$". More specifically, if $mathbf{b}$ is in the column space of $mathbf{A}$, then there is a solution.



          So is there a way that we can tell whether $mathbf{A}$ behaves like zero in some directions? Yes, it is the determinant! If $det(mathbf{A})neq 0$ then $mathbf{Ax} = mathbf{b}$ always has a solution. However if, $det(mathbf{A}) = 0$ then $mathbf{Ax} = mathbf{b}$ may or may not have a solution depending on $mathbf{b}$ and if there is one, then there are an infinite number of solutions.






          share|cite|improve this answer









          $endgroup$





















            2












            $begingroup$

            One way to treat define the determinant which makes clear the relation between all the various notions you mentioned is as follows:



            Given a vector space $E$ of dimmension $n$ over the field $K$ and a basis $B=(b_1,...,b_n)$ of $E$, the determinant is the unique (nonzero) alternating multilinear $n$-form $phi$ of $E$ which satisfies $phi(b_1,...,b_n)=1$.



            This simply means that the determinant is a function $phi$ which takes a tuple $(x_1,...,x_n)$ of $n$ vectors of $E$ and returns a scalar from the field $K$, such that



            (1) $phi$ is linear in each of the $n$ variables $(x_1,...,x_n)$ (it's "multilinear")



            (2) if two of the $x_i$'s are equal, then $phi(x_1,...,x_n)=0$ ($phi$ is "alternating")



            (3) It turns out that the set of functions $phi$ satisfying the two above properties are all multiples of eachother. So we choose a basis $B$ of $e$ and say that the determinant is the function $phi$ satisfying the above properties which maps $B$ to $1$.



            Of course it's not immediately obvious that such a function $phi$ exists and is unique!



            To simplify slightly we will take the vector space $E$ to be $K^n$ and the basis $B$ to be the canonical basis.



            It turns out that the determinant satisfies the miraculous property that $det(x_1,...,x_n) neq 0$ if and only if $(x_1,...,x_n)$ is a basis.



            Now... given $n$ vectors $x_1,...,x_n$ such that for the coordinates in the basis $B$ of $x_i$ are $(a_{i,1},...,a_{i,n})$, the determinant of the $n$-vectors $x_1,...,x_n$ can be shown to be equal to



            $sum_{sigma in S_n} sgn(sigma)a_{1,sigma(1)}...a_{n,sigma(n)}$



            which should be familiar to you as the expression for the determinant in terms of permutations. Here $S_n$ is the symmetric group, i.e the set of permutations of ${1,2,..,n}$ and $sgn(sigma)$ is the signature of the permutation $sigma$.



            To make the link between the determinant of a set of $n$ vectors to the determinant of a matrix, just note that the matrix $A=(a_{i,j})$ is exactly them matrix whose column vectors are $x_1,...,x_n$.



            Thus when we take the determinant of a matrix, what we are really doing is evaluating a function in terms of the $n$ column vectors. We said earlier that this function is nonzero if and only if the $n$ vectors form a basis - in other words, if and only if the matrix is of full rank, i.e iff its invertible.



            So the abstract definition the determinant as a function which maps a set of vectors to the scalar field (while obeying some nice properties like linearity) is equivalent to a function from matrices to the scalar field which is nonzero exactly when the matrix is invertible. Moreover, this function turns out to be multiplicative! (Consequently, the restriction of this function to the set of invertible matrices gives is a group homomorphism from $(Gl_n(K), times)$ to $(K/{0},*)$.



            The expression of the determinant of a matrix in terms of permutations can be used to derive many of the nice properties you are familiar with, for example




            • a matrix and its transpose have the same det


            • det of a triangular matrix is the product of the diagonal elements


            • the Laplace-formula a.k.a cofactor expansion which tells you how to calculate the determinant in terms of a weighted sum of determinants of submatrices:



            $det(A)=sum_{i=1}^{n}(-1)^{i+j}a_{i,j}Delta_{i,j}$



            where $Delta_{i,j}$ is the determinant of the matrix obtained from $A$ by removing the row $i$ and the column $j$, known as the minor $(i,j)$.






            share|cite|improve this answer











            $endgroup$





















              0












              $begingroup$

              Imagine a completely general system of equations



              $$a_{11} x_1 + a_{12} x_2 + a_{13} x_3 = b_1$$
              $$a_{21} x_1 + a_{22} x_2 + a_{23} x_3 = b_2$$
              $$a_{31} x_1 + a_{32} x_2 + a_{33} x_3 = b_3$$



              If we solve for the variables $x_i$ in terms of the other variables and write the results in lowest terms, we'll see that the expressions for each $x_i$ all have the same functions of $a_{ij}$ in the denominator. (Say that we work over the integers.) This expression is (up to a unit) the determinant of the system.



              If you pick some systematic way of solving $n times n$ systems, say Gaussian elimination, you can use it to crank out a formula for this determinant.



              I think this is a lot more natural than the other approaches because you start with something straightforward and common like a system of linear equations, then you put your head down and solve it, and out pops this notion.



              Of course this only gives you the answer up to a sign, but this actually makes sense, because there's an arbitrary choice of sign going in.



              Garibaldi has a paper that presents this approach and some related ones, entitled The determinant and characteristic polynomial are not ad hoc constructions. (To formalize this you want to bring in a little ring theory so that you have formal notions of indeterminates and so forth.)






              share|cite|improve this answer











              $endgroup$












                protected by Community Apr 20 '15 at 22:34



                Thank you for your interest in this question.
                Because it has attracted low-quality or spam answers that had to be removed, posting an answer now requires 10 reputation on this site (the association bonus does not count).



                Would you like to answer one of these unanswered questions instead?














                13 Answers
                13






                active

                oldest

                votes








                13 Answers
                13






                active

                oldest

                votes









                active

                oldest

                votes






                active

                oldest

                votes









                344












                $begingroup$

                Your trouble with determinants is pretty common. They’re a hard thing to teach well, too, for two main reasons that I can see: the formulas you learn for computing them are messy and complicated, and there’s no “natural” way to interpret the value of the determinant, the way it’s easy to interpret the derivatives you do in calculus at first as the slope of the tangent line. It’s hard to believe things like the invertibility condition you’ve stated when it’s not even clear what the numbers mean and where they come from.



                Rather than show that the many usual definitions are all the same by comparing them to each other, I’m going to state some general properties of the determinant that I claim are enough to specify uniquely what number you should get when you put in a given matrix. Then it’s not too bad to check that all of the definitions for determinant that you’ve seen satisfy those properties I’ll state.



                The first thing to think about if you want an “abstract” definition of the determinant to unify all those others is that it’s not an array of numbers with bars on the side. What we’re really looking for is a function that takes N vectors (the N columns of the matrix) and returns a number. Let’s assume we’re working with real numbers for now.



                Remember how those operations you mentioned change the value of the determinant?




                1. Switching two rows or columns changes the sign.


                2. Multiplying one row by a constant multiplies the whole determinant by that constant.


                3. The general fact that number two draws from: the determinant is linear in each row. That is, if you think of it as a function $det: mathbb{R}^{n^2} rightarrow mathbb{R}$, then $$ det(a vec v_1 +b vec w_1 , vec v_2 ,ldots,vec v_n ) = a det(vec v_1,vec v_2,ldots,vec v_n) + b det(vec w_1, vec v_2, ldots,vec v_n),$$ and the corresponding condition in each other slot.


                4. The determinant of the identity matrix $I$ is $1$.



                I claim that these facts are enough to define a unique function that takes in N vectors (each of length N) and returns a real number, the determinant of the matrix given by those vectors. I won’t prove that, but I’ll show you how it helps with some other interpretations of the determinant.



                In particular, there’s a nice geometric way to think of a determinant. Consider the unit cube in N dimensional space: the set of vectors of length N with coordinates 0 or 1 in each spot. The determinant of the linear transformation (matrix) T is the signed volume of the region gotten by applying T to the unit cube. (Don’t worry too much if you don’t know what the “signed” part means, for now).



                How does that follow from our abstract definition?



                Well, if you apply the identity to the unit cube, you get back the unit cube. And the volume of the unit cube is 1.



                If you stretch the cube by a constant factor in one direction only, the new volume is that constant. And if you stack two blocks together aligned on the same direction, their combined volume is the sum of their volumes: this all shows that the signed volume we have is linear in each coordinate when considered as a function of the input vectors.



                Finally, when you switch two of the vectors that define the unit cube, you flip the orientation. (Again, this is something to come back to later if you don’t know what that means).



                So there are ways to think about the determinant that aren’t symbol-pushing. If you’ve studied multivariable calculus, you could think about, with this geometric definition of determinant, why determinants (the Jacobian) pop up when we change coordinates doing integration. Hint: a derivative is a linear approximations of the associated function, and consider a “differential volume element” in your starting coordinate system.



                It’s not too much work to check that the area of the parallelogram formed by vectors $(a,b)$ and $(c,d)$ is $Big|{}^{a;b}_{c;d}Big|$
                either: you might try that to get a sense for things.






                share|cite|improve this answer











                $endgroup$









                • 22




                  $begingroup$
                  Very nice explanation. I think you should clarify what you mean by "the determinate is linear in each row" though
                  $endgroup$
                  – Casebash
                  Jul 25 '10 at 3:07






                • 2




                  $begingroup$
                  Great answer. We were taught the determinant as the generalized volume function in our algebra class.
                  $endgroup$
                  – Neil G
                  Aug 28 '10 at 9:26






                • 15




                  $begingroup$
                  Just out of curiosity, who are you talking to with the first sentence? Didn't you ask the question?!? Either way, I point students to this Q (&A) all the time for determinant help.
                  $endgroup$
                  – The Chaz 2.0
                  Apr 18 '12 at 2:18






                • 9




                  $begingroup$
                  @TheChaz this question was asked near the beginnings of Math.SE, when there was a need to populate the site with questions before it was opened up to 'the public'. In any case, answering your own questions is explicitly encouraged nowadays.
                  $endgroup$
                  – Chris Taylor
                  May 24 '12 at 7:51






                • 12




                  $begingroup$
                  To see that the geometric interpretation (volume of the image of cube) satisfies the multilinearity property it is not enough to stack two "aligned" blocks, this is again the multiplication of a column by a scalar. You should deal with two "not aligned" blocks produced by changing a single column vector, and see that the sum of their volumes is the volume of the block obtained putting the sum of the two vectors. This is not so easy to catch...
                  $endgroup$
                  – Emanuele Paolini
                  Feb 20 '13 at 9:19
















                344












                $begingroup$

                Your trouble with determinants is pretty common. They’re a hard thing to teach well, too, for two main reasons that I can see: the formulas you learn for computing them are messy and complicated, and there’s no “natural” way to interpret the value of the determinant, the way it’s easy to interpret the derivatives you do in calculus at first as the slope of the tangent line. It’s hard to believe things like the invertibility condition you’ve stated when it’s not even clear what the numbers mean and where they come from.



                Rather than show that the many usual definitions are all the same by comparing them to each other, I’m going to state some general properties of the determinant that I claim are enough to specify uniquely what number you should get when you put in a given matrix. Then it’s not too bad to check that all of the definitions for determinant that you’ve seen satisfy those properties I’ll state.



                The first thing to think about if you want an “abstract” definition of the determinant to unify all those others is that it’s not an array of numbers with bars on the side. What we’re really looking for is a function that takes N vectors (the N columns of the matrix) and returns a number. Let’s assume we’re working with real numbers for now.



                Remember how those operations you mentioned change the value of the determinant?




                1. Switching two rows or columns changes the sign.


                2. Multiplying one row by a constant multiplies the whole determinant by that constant.


                3. The general fact that number two draws from: the determinant is linear in each row. That is, if you think of it as a function $det: mathbb{R}^{n^2} rightarrow mathbb{R}$, then $$ det(a vec v_1 +b vec w_1 , vec v_2 ,ldots,vec v_n ) = a det(vec v_1,vec v_2,ldots,vec v_n) + b det(vec w_1, vec v_2, ldots,vec v_n),$$ and the corresponding condition in each other slot.


                4. The determinant of the identity matrix $I$ is $1$.



                I claim that these facts are enough to define a unique function that takes in N vectors (each of length N) and returns a real number, the determinant of the matrix given by those vectors. I won’t prove that, but I’ll show you how it helps with some other interpretations of the determinant.



                In particular, there’s a nice geometric way to think of a determinant. Consider the unit cube in N dimensional space: the set of vectors of length N with coordinates 0 or 1 in each spot. The determinant of the linear transformation (matrix) T is the signed volume of the region gotten by applying T to the unit cube. (Don’t worry too much if you don’t know what the “signed” part means, for now).



                How does that follow from our abstract definition?



                Well, if you apply the identity to the unit cube, you get back the unit cube. And the volume of the unit cube is 1.



                If you stretch the cube by a constant factor in one direction only, the new volume is that constant. And if you stack two blocks together aligned on the same direction, their combined volume is the sum of their volumes: this all shows that the signed volume we have is linear in each coordinate when considered as a function of the input vectors.



                Finally, when you switch two of the vectors that define the unit cube, you flip the orientation. (Again, this is something to come back to later if you don’t know what that means).



                So there are ways to think about the determinant that aren’t symbol-pushing. If you’ve studied multivariable calculus, you could think about, with this geometric definition of determinant, why determinants (the Jacobian) pop up when we change coordinates doing integration. Hint: a derivative is a linear approximations of the associated function, and consider a “differential volume element” in your starting coordinate system.



                It’s not too much work to check that the area of the parallelogram formed by vectors $(a,b)$ and $(c,d)$ is $Big|{}^{a;b}_{c;d}Big|$
                either: you might try that to get a sense for things.






                share|cite|improve this answer











                $endgroup$









                • 22




                  $begingroup$
                  Very nice explanation. I think you should clarify what you mean by "the determinate is linear in each row" though
                  $endgroup$
                  – Casebash
                  Jul 25 '10 at 3:07






                • 2




                  $begingroup$
                  Great answer. We were taught the determinant as the generalized volume function in our algebra class.
                  $endgroup$
                  – Neil G
                  Aug 28 '10 at 9:26






                • 15




                  $begingroup$
                  Just out of curiosity, who are you talking to with the first sentence? Didn't you ask the question?!? Either way, I point students to this Q (&A) all the time for determinant help.
                  $endgroup$
                  – The Chaz 2.0
                  Apr 18 '12 at 2:18






                • 9




                  $begingroup$
                  @TheChaz this question was asked near the beginnings of Math.SE, when there was a need to populate the site with questions before it was opened up to 'the public'. In any case, answering your own questions is explicitly encouraged nowadays.
                  $endgroup$
                  – Chris Taylor
                  May 24 '12 at 7:51






                • 12




                  $begingroup$
                  To see that the geometric interpretation (volume of the image of cube) satisfies the multilinearity property it is not enough to stack two "aligned" blocks, this is again the multiplication of a column by a scalar. You should deal with two "not aligned" blocks produced by changing a single column vector, and see that the sum of their volumes is the volume of the block obtained putting the sum of the two vectors. This is not so easy to catch...
                  $endgroup$
                  – Emanuele Paolini
                  Feb 20 '13 at 9:19














                344












                344








                344





                $begingroup$

                Your trouble with determinants is pretty common. They’re a hard thing to teach well, too, for two main reasons that I can see: the formulas you learn for computing them are messy and complicated, and there’s no “natural” way to interpret the value of the determinant, the way it’s easy to interpret the derivatives you do in calculus at first as the slope of the tangent line. It’s hard to believe things like the invertibility condition you’ve stated when it’s not even clear what the numbers mean and where they come from.



                Rather than show that the many usual definitions are all the same by comparing them to each other, I’m going to state some general properties of the determinant that I claim are enough to specify uniquely what number you should get when you put in a given matrix. Then it’s not too bad to check that all of the definitions for determinant that you’ve seen satisfy those properties I’ll state.



                The first thing to think about if you want an “abstract” definition of the determinant to unify all those others is that it’s not an array of numbers with bars on the side. What we’re really looking for is a function that takes N vectors (the N columns of the matrix) and returns a number. Let’s assume we’re working with real numbers for now.



                Remember how those operations you mentioned change the value of the determinant?




                1. Switching two rows or columns changes the sign.


                2. Multiplying one row by a constant multiplies the whole determinant by that constant.


                3. The general fact that number two draws from: the determinant is linear in each row. That is, if you think of it as a function $det: mathbb{R}^{n^2} rightarrow mathbb{R}$, then $$ det(a vec v_1 +b vec w_1 , vec v_2 ,ldots,vec v_n ) = a det(vec v_1,vec v_2,ldots,vec v_n) + b det(vec w_1, vec v_2, ldots,vec v_n),$$ and the corresponding condition in each other slot.


                4. The determinant of the identity matrix $I$ is $1$.



                I claim that these facts are enough to define a unique function that takes in N vectors (each of length N) and returns a real number, the determinant of the matrix given by those vectors. I won’t prove that, but I’ll show you how it helps with some other interpretations of the determinant.



                In particular, there’s a nice geometric way to think of a determinant. Consider the unit cube in N dimensional space: the set of vectors of length N with coordinates 0 or 1 in each spot. The determinant of the linear transformation (matrix) T is the signed volume of the region gotten by applying T to the unit cube. (Don’t worry too much if you don’t know what the “signed” part means, for now).



                How does that follow from our abstract definition?



                Well, if you apply the identity to the unit cube, you get back the unit cube. And the volume of the unit cube is 1.



                If you stretch the cube by a constant factor in one direction only, the new volume is that constant. And if you stack two blocks together aligned on the same direction, their combined volume is the sum of their volumes: this all shows that the signed volume we have is linear in each coordinate when considered as a function of the input vectors.



                Finally, when you switch two of the vectors that define the unit cube, you flip the orientation. (Again, this is something to come back to later if you don’t know what that means).



                So there are ways to think about the determinant that aren’t symbol-pushing. If you’ve studied multivariable calculus, you could think about, with this geometric definition of determinant, why determinants (the Jacobian) pop up when we change coordinates doing integration. Hint: a derivative is a linear approximations of the associated function, and consider a “differential volume element” in your starting coordinate system.



                It’s not too much work to check that the area of the parallelogram formed by vectors $(a,b)$ and $(c,d)$ is $Big|{}^{a;b}_{c;d}Big|$
                either: you might try that to get a sense for things.






                share|cite|improve this answer











                $endgroup$



                Your trouble with determinants is pretty common. They’re a hard thing to teach well, too, for two main reasons that I can see: the formulas you learn for computing them are messy and complicated, and there’s no “natural” way to interpret the value of the determinant, the way it’s easy to interpret the derivatives you do in calculus at first as the slope of the tangent line. It’s hard to believe things like the invertibility condition you’ve stated when it’s not even clear what the numbers mean and where they come from.



                Rather than show that the many usual definitions are all the same by comparing them to each other, I’m going to state some general properties of the determinant that I claim are enough to specify uniquely what number you should get when you put in a given matrix. Then it’s not too bad to check that all of the definitions for determinant that you’ve seen satisfy those properties I’ll state.



                The first thing to think about if you want an “abstract” definition of the determinant to unify all those others is that it’s not an array of numbers with bars on the side. What we’re really looking for is a function that takes N vectors (the N columns of the matrix) and returns a number. Let’s assume we’re working with real numbers for now.



                Remember how those operations you mentioned change the value of the determinant?




                1. Switching two rows or columns changes the sign.


                2. Multiplying one row by a constant multiplies the whole determinant by that constant.


                3. The general fact that number two draws from: the determinant is linear in each row. That is, if you think of it as a function $det: mathbb{R}^{n^2} rightarrow mathbb{R}$, then $$ det(a vec v_1 +b vec w_1 , vec v_2 ,ldots,vec v_n ) = a det(vec v_1,vec v_2,ldots,vec v_n) + b det(vec w_1, vec v_2, ldots,vec v_n),$$ and the corresponding condition in each other slot.


                4. The determinant of the identity matrix $I$ is $1$.



                I claim that these facts are enough to define a unique function that takes in N vectors (each of length N) and returns a real number, the determinant of the matrix given by those vectors. I won’t prove that, but I’ll show you how it helps with some other interpretations of the determinant.



                In particular, there’s a nice geometric way to think of a determinant. Consider the unit cube in N dimensional space: the set of vectors of length N with coordinates 0 or 1 in each spot. The determinant of the linear transformation (matrix) T is the signed volume of the region gotten by applying T to the unit cube. (Don’t worry too much if you don’t know what the “signed” part means, for now).



                How does that follow from our abstract definition?



                Well, if you apply the identity to the unit cube, you get back the unit cube. And the volume of the unit cube is 1.



                If you stretch the cube by a constant factor in one direction only, the new volume is that constant. And if you stack two blocks together aligned on the same direction, their combined volume is the sum of their volumes: this all shows that the signed volume we have is linear in each coordinate when considered as a function of the input vectors.



                Finally, when you switch two of the vectors that define the unit cube, you flip the orientation. (Again, this is something to come back to later if you don’t know what that means).



                So there are ways to think about the determinant that aren’t symbol-pushing. If you’ve studied multivariable calculus, you could think about, with this geometric definition of determinant, why determinants (the Jacobian) pop up when we change coordinates doing integration. Hint: a derivative is a linear approximations of the associated function, and consider a “differential volume element” in your starting coordinate system.



                It’s not too much work to check that the area of the parallelogram formed by vectors $(a,b)$ and $(c,d)$ is $Big|{}^{a;b}_{c;d}Big|$
                either: you might try that to get a sense for things.







                share|cite|improve this answer














                share|cite|improve this answer



                share|cite|improve this answer








                edited Dec 22 '16 at 17:52


























                community wiki





                6 revs, 5 users 74%
                Jamie Banks









                • 22




                  $begingroup$
                  Very nice explanation. I think you should clarify what you mean by "the determinate is linear in each row" though
                  $endgroup$
                  – Casebash
                  Jul 25 '10 at 3:07






                • 2




                  $begingroup$
                  Great answer. We were taught the determinant as the generalized volume function in our algebra class.
                  $endgroup$
                  – Neil G
                  Aug 28 '10 at 9:26






                • 15




                  $begingroup$
                  Just out of curiosity, who are you talking to with the first sentence? Didn't you ask the question?!? Either way, I point students to this Q (&A) all the time for determinant help.
                  $endgroup$
                  – The Chaz 2.0
                  Apr 18 '12 at 2:18






                • 9




                  $begingroup$
                  @TheChaz this question was asked near the beginnings of Math.SE, when there was a need to populate the site with questions before it was opened up to 'the public'. In any case, answering your own questions is explicitly encouraged nowadays.
                  $endgroup$
                  – Chris Taylor
                  May 24 '12 at 7:51






                • 12




                  $begingroup$
                  To see that the geometric interpretation (volume of the image of cube) satisfies the multilinearity property it is not enough to stack two "aligned" blocks, this is again the multiplication of a column by a scalar. You should deal with two "not aligned" blocks produced by changing a single column vector, and see that the sum of their volumes is the volume of the block obtained putting the sum of the two vectors. This is not so easy to catch...
                  $endgroup$
                  – Emanuele Paolini
                  Feb 20 '13 at 9:19














                • 22




                  $begingroup$
                  Very nice explanation. I think you should clarify what you mean by "the determinate is linear in each row" though
                  $endgroup$
                  – Casebash
                  Jul 25 '10 at 3:07






                • 2




                  $begingroup$
                  Great answer. We were taught the determinant as the generalized volume function in our algebra class.
                  $endgroup$
                  – Neil G
                  Aug 28 '10 at 9:26






                • 15




                  $begingroup$
                  Just out of curiosity, who are you talking to with the first sentence? Didn't you ask the question?!? Either way, I point students to this Q (&A) all the time for determinant help.
                  $endgroup$
                  – The Chaz 2.0
                  Apr 18 '12 at 2:18






                • 9




                  $begingroup$
                  @TheChaz this question was asked near the beginnings of Math.SE, when there was a need to populate the site with questions before it was opened up to 'the public'. In any case, answering your own questions is explicitly encouraged nowadays.
                  $endgroup$
                  – Chris Taylor
                  May 24 '12 at 7:51






                • 12




                  $begingroup$
                  To see that the geometric interpretation (volume of the image of cube) satisfies the multilinearity property it is not enough to stack two "aligned" blocks, this is again the multiplication of a column by a scalar. You should deal with two "not aligned" blocks produced by changing a single column vector, and see that the sum of their volumes is the volume of the block obtained putting the sum of the two vectors. This is not so easy to catch...
                  $endgroup$
                  – Emanuele Paolini
                  Feb 20 '13 at 9:19








                22




                22




                $begingroup$
                Very nice explanation. I think you should clarify what you mean by "the determinate is linear in each row" though
                $endgroup$
                – Casebash
                Jul 25 '10 at 3:07




                $begingroup$
                Very nice explanation. I think you should clarify what you mean by "the determinate is linear in each row" though
                $endgroup$
                – Casebash
                Jul 25 '10 at 3:07




                2




                2




                $begingroup$
                Great answer. We were taught the determinant as the generalized volume function in our algebra class.
                $endgroup$
                – Neil G
                Aug 28 '10 at 9:26




                $begingroup$
                Great answer. We were taught the determinant as the generalized volume function in our algebra class.
                $endgroup$
                – Neil G
                Aug 28 '10 at 9:26




                15




                15




                $begingroup$
                Just out of curiosity, who are you talking to with the first sentence? Didn't you ask the question?!? Either way, I point students to this Q (&A) all the time for determinant help.
                $endgroup$
                – The Chaz 2.0
                Apr 18 '12 at 2:18




                $begingroup$
                Just out of curiosity, who are you talking to with the first sentence? Didn't you ask the question?!? Either way, I point students to this Q (&A) all the time for determinant help.
                $endgroup$
                – The Chaz 2.0
                Apr 18 '12 at 2:18




                9




                9




                $begingroup$
                @TheChaz this question was asked near the beginnings of Math.SE, when there was a need to populate the site with questions before it was opened up to 'the public'. In any case, answering your own questions is explicitly encouraged nowadays.
                $endgroup$
                – Chris Taylor
                May 24 '12 at 7:51




                $begingroup$
                @TheChaz this question was asked near the beginnings of Math.SE, when there was a need to populate the site with questions before it was opened up to 'the public'. In any case, answering your own questions is explicitly encouraged nowadays.
                $endgroup$
                – Chris Taylor
                May 24 '12 at 7:51




                12




                12




                $begingroup$
                To see that the geometric interpretation (volume of the image of cube) satisfies the multilinearity property it is not enough to stack two "aligned" blocks, this is again the multiplication of a column by a scalar. You should deal with two "not aligned" blocks produced by changing a single column vector, and see that the sum of their volumes is the volume of the block obtained putting the sum of the two vectors. This is not so easy to catch...
                $endgroup$
                – Emanuele Paolini
                Feb 20 '13 at 9:19




                $begingroup$
                To see that the geometric interpretation (volume of the image of cube) satisfies the multilinearity property it is not enough to stack two "aligned" blocks, this is again the multiplication of a column by a scalar. You should deal with two "not aligned" blocks produced by changing a single column vector, and see that the sum of their volumes is the volume of the block obtained putting the sum of the two vectors. This is not so easy to catch...
                $endgroup$
                – Emanuele Paolini
                Feb 20 '13 at 9:19











                194












                $begingroup$

                You could think of a determinant as a volume. Think of the columns of the matrix as vectors at the origin forming the edges of a skewed box. The determinant gives the volume of that box. For example, in 2 dimensions, the columns of the matrix are the edges of a rhombus.



                You can derive the algebraic properties from this geometrical interpretation. For example, if two of the columns are linearly dependent, your box is missing a dimension and so it's been flattened to have zero volume.






                share|cite|improve this answer











                $endgroup$









                • 51




                  $begingroup$
                  If I may, I would add to this answer (which I think is a very good one) in two minor aspects. First, a determinant also has a sign, so we want the concept of oriented volume. (This is somewhat tricky, but definitely important, so you might as well have it in mind when you're learning about "right hand rules" and such.) Second, I think better than a volume is thinking of the determinant as the multiplicative change in volume of a parallelopiped under the linear transformation. (Of course you can always take the first one to be the unit n-cube and say that you are just dividing by one.)
                  $endgroup$
                  – Pete L. Clark
                  Jul 28 '10 at 20:08






                • 8




                  $begingroup$
                  +1: I like this answer because there is a direct link to some application in physics: In special relativity we are talking of the conservation of space-time-volume, which means that the determinant of the transformation matrix is const. 1
                  $endgroup$
                  – vonjd
                  Jan 16 '11 at 10:22
















                194












                $begingroup$

                You could think of a determinant as a volume. Think of the columns of the matrix as vectors at the origin forming the edges of a skewed box. The determinant gives the volume of that box. For example, in 2 dimensions, the columns of the matrix are the edges of a rhombus.



                You can derive the algebraic properties from this geometrical interpretation. For example, if two of the columns are linearly dependent, your box is missing a dimension and so it's been flattened to have zero volume.






                share|cite|improve this answer











                $endgroup$









                • 51




                  $begingroup$
                  If I may, I would add to this answer (which I think is a very good one) in two minor aspects. First, a determinant also has a sign, so we want the concept of oriented volume. (This is somewhat tricky, but definitely important, so you might as well have it in mind when you're learning about "right hand rules" and such.) Second, I think better than a volume is thinking of the determinant as the multiplicative change in volume of a parallelopiped under the linear transformation. (Of course you can always take the first one to be the unit n-cube and say that you are just dividing by one.)
                  $endgroup$
                  – Pete L. Clark
                  Jul 28 '10 at 20:08






                • 8




                  $begingroup$
                  +1: I like this answer because there is a direct link to some application in physics: In special relativity we are talking of the conservation of space-time-volume, which means that the determinant of the transformation matrix is const. 1
                  $endgroup$
                  – vonjd
                  Jan 16 '11 at 10:22














                194












                194








                194





                $begingroup$

                You could think of a determinant as a volume. Think of the columns of the matrix as vectors at the origin forming the edges of a skewed box. The determinant gives the volume of that box. For example, in 2 dimensions, the columns of the matrix are the edges of a rhombus.



                You can derive the algebraic properties from this geometrical interpretation. For example, if two of the columns are linearly dependent, your box is missing a dimension and so it's been flattened to have zero volume.






                share|cite|improve this answer











                $endgroup$



                You could think of a determinant as a volume. Think of the columns of the matrix as vectors at the origin forming the edges of a skewed box. The determinant gives the volume of that box. For example, in 2 dimensions, the columns of the matrix are the edges of a rhombus.



                You can derive the algebraic properties from this geometrical interpretation. For example, if two of the columns are linearly dependent, your box is missing a dimension and so it's been flattened to have zero volume.







                share|cite|improve this answer














                share|cite|improve this answer



                share|cite|improve this answer








                edited May 13 '13 at 7:14









                MJD

                47.6k29215397




                47.6k29215397










                answered Jul 28 '10 at 19:05









                John D. CookJohn D. Cook

                5,59222033




                5,59222033








                • 51




                  $begingroup$
                  If I may, I would add to this answer (which I think is a very good one) in two minor aspects. First, a determinant also has a sign, so we want the concept of oriented volume. (This is somewhat tricky, but definitely important, so you might as well have it in mind when you're learning about "right hand rules" and such.) Second, I think better than a volume is thinking of the determinant as the multiplicative change in volume of a parallelopiped under the linear transformation. (Of course you can always take the first one to be the unit n-cube and say that you are just dividing by one.)
                  $endgroup$
                  – Pete L. Clark
                  Jul 28 '10 at 20:08






                • 8




                  $begingroup$
                  +1: I like this answer because there is a direct link to some application in physics: In special relativity we are talking of the conservation of space-time-volume, which means that the determinant of the transformation matrix is const. 1
                  $endgroup$
                  – vonjd
                  Jan 16 '11 at 10:22














                • 51




                  $begingroup$
                  If I may, I would add to this answer (which I think is a very good one) in two minor aspects. First, a determinant also has a sign, so we want the concept of oriented volume. (This is somewhat tricky, but definitely important, so you might as well have it in mind when you're learning about "right hand rules" and such.) Second, I think better than a volume is thinking of the determinant as the multiplicative change in volume of a parallelopiped under the linear transformation. (Of course you can always take the first one to be the unit n-cube and say that you are just dividing by one.)
                  $endgroup$
                  – Pete L. Clark
                  Jul 28 '10 at 20:08






                • 8




                  $begingroup$
                  +1: I like this answer because there is a direct link to some application in physics: In special relativity we are talking of the conservation of space-time-volume, which means that the determinant of the transformation matrix is const. 1
                  $endgroup$
                  – vonjd
                  Jan 16 '11 at 10:22








                51




                51




                $begingroup$
                If I may, I would add to this answer (which I think is a very good one) in two minor aspects. First, a determinant also has a sign, so we want the concept of oriented volume. (This is somewhat tricky, but definitely important, so you might as well have it in mind when you're learning about "right hand rules" and such.) Second, I think better than a volume is thinking of the determinant as the multiplicative change in volume of a parallelopiped under the linear transformation. (Of course you can always take the first one to be the unit n-cube and say that you are just dividing by one.)
                $endgroup$
                – Pete L. Clark
                Jul 28 '10 at 20:08




                $begingroup$
                If I may, I would add to this answer (which I think is a very good one) in two minor aspects. First, a determinant also has a sign, so we want the concept of oriented volume. (This is somewhat tricky, but definitely important, so you might as well have it in mind when you're learning about "right hand rules" and such.) Second, I think better than a volume is thinking of the determinant as the multiplicative change in volume of a parallelopiped under the linear transformation. (Of course you can always take the first one to be the unit n-cube and say that you are just dividing by one.)
                $endgroup$
                – Pete L. Clark
                Jul 28 '10 at 20:08




                8




                8




                $begingroup$
                +1: I like this answer because there is a direct link to some application in physics: In special relativity we are talking of the conservation of space-time-volume, which means that the determinant of the transformation matrix is const. 1
                $endgroup$
                – vonjd
                Jan 16 '11 at 10:22




                $begingroup$
                +1: I like this answer because there is a direct link to some application in physics: In special relativity we are talking of the conservation of space-time-volume, which means that the determinant of the transformation matrix is const. 1
                $endgroup$
                – vonjd
                Jan 16 '11 at 10:22











                97












                $begingroup$

                In addition to the answers, above, the determinant is a function from the set of square matrices into the real numbers that preserves the operation of multiplication:
                begin{equation}det(AB) = det(A)det(B) end{equation}
                and so it carries $some$ information about square matrices into the much more familiar set of real numbers.



                Some examples:



                The determinant function maps the identity matrix $I$ to the identity element of the real numbers ($det(I) = 1$.)



                Which real number does not have a multiplicative inverse? The number 0. So which square matrices do not have multiplicative inverses? Those which are mapped to 0 by the determinant function.



                What is the determinant of the inverse of a matrix? The inverse of the determinant, of course. (Etc.)



                This "operation preserving" property of the determinant explains some of the value of the determinant function and provides a certain level of "intuition" for me in working with matrices.






                share|cite|improve this answer











                $endgroup$









                • 7




                  $begingroup$
                  +1 for including the questions. Many of them. Good ones. Especially the "So which square matrices do not have multiplicative inverses?" pair. And for featuring a nice doggy in your portrait!
                  $endgroup$
                  – n611x007
                  Jan 30 '13 at 17:40


















                97












                $begingroup$

                In addition to the answers, above, the determinant is a function from the set of square matrices into the real numbers that preserves the operation of multiplication:
                begin{equation}det(AB) = det(A)det(B) end{equation}
                and so it carries $some$ information about square matrices into the much more familiar set of real numbers.



                Some examples:



                The determinant function maps the identity matrix $I$ to the identity element of the real numbers ($det(I) = 1$.)



                Which real number does not have a multiplicative inverse? The number 0. So which square matrices do not have multiplicative inverses? Those which are mapped to 0 by the determinant function.



                What is the determinant of the inverse of a matrix? The inverse of the determinant, of course. (Etc.)



                This "operation preserving" property of the determinant explains some of the value of the determinant function and provides a certain level of "intuition" for me in working with matrices.






                share|cite|improve this answer











                $endgroup$









                • 7




                  $begingroup$
                  +1 for including the questions. Many of them. Good ones. Especially the "So which square matrices do not have multiplicative inverses?" pair. And for featuring a nice doggy in your portrait!
                  $endgroup$
                  – n611x007
                  Jan 30 '13 at 17:40
















                97












                97








                97





                $begingroup$

                In addition to the answers, above, the determinant is a function from the set of square matrices into the real numbers that preserves the operation of multiplication:
                begin{equation}det(AB) = det(A)det(B) end{equation}
                and so it carries $some$ information about square matrices into the much more familiar set of real numbers.



                Some examples:



                The determinant function maps the identity matrix $I$ to the identity element of the real numbers ($det(I) = 1$.)



                Which real number does not have a multiplicative inverse? The number 0. So which square matrices do not have multiplicative inverses? Those which are mapped to 0 by the determinant function.



                What is the determinant of the inverse of a matrix? The inverse of the determinant, of course. (Etc.)



                This "operation preserving" property of the determinant explains some of the value of the determinant function and provides a certain level of "intuition" for me in working with matrices.






                share|cite|improve this answer











                $endgroup$



                In addition to the answers, above, the determinant is a function from the set of square matrices into the real numbers that preserves the operation of multiplication:
                begin{equation}det(AB) = det(A)det(B) end{equation}
                and so it carries $some$ information about square matrices into the much more familiar set of real numbers.



                Some examples:



                The determinant function maps the identity matrix $I$ to the identity element of the real numbers ($det(I) = 1$.)



                Which real number does not have a multiplicative inverse? The number 0. So which square matrices do not have multiplicative inverses? Those which are mapped to 0 by the determinant function.



                What is the determinant of the inverse of a matrix? The inverse of the determinant, of course. (Etc.)



                This "operation preserving" property of the determinant explains some of the value of the determinant function and provides a certain level of "intuition" for me in working with matrices.







                share|cite|improve this answer














                share|cite|improve this answer



                share|cite|improve this answer








                edited Jun 13 '16 at 18:11









                Mars

                300116




                300116










                answered Jan 15 '11 at 21:37









                KenWSmithKenWSmith

                1,3981010




                1,3981010








                • 7




                  $begingroup$
                  +1 for including the questions. Many of them. Good ones. Especially the "So which square matrices do not have multiplicative inverses?" pair. And for featuring a nice doggy in your portrait!
                  $endgroup$
                  – n611x007
                  Jan 30 '13 at 17:40
















                • 7




                  $begingroup$
                  +1 for including the questions. Many of them. Good ones. Especially the "So which square matrices do not have multiplicative inverses?" pair. And for featuring a nice doggy in your portrait!
                  $endgroup$
                  – n611x007
                  Jan 30 '13 at 17:40










                7




                7




                $begingroup$
                +1 for including the questions. Many of them. Good ones. Especially the "So which square matrices do not have multiplicative inverses?" pair. And for featuring a nice doggy in your portrait!
                $endgroup$
                – n611x007
                Jan 30 '13 at 17:40






                $begingroup$
                +1 for including the questions. Many of them. Good ones. Especially the "So which square matrices do not have multiplicative inverses?" pair. And for featuring a nice doggy in your portrait!
                $endgroup$
                – n611x007
                Jan 30 '13 at 17:40













                37












                $begingroup$

                Here is a recording of my lecture on the geometric definition of determinants:



                Geometric definition of determinants



                It has elements from the answers by Jamie Banks and John Cook, and goes into details in a leisurely manner.






                share|cite|improve this answer











                $endgroup$









                • 1




                  $begingroup$
                  +1 for a detailed explaination video. You should put the disclosure that you are the lecturer. At first I thought you are a student in the class. By the way, I would like to see you make a full lectures list about linear algebra. It would be very nice.
                  $endgroup$
                  – Anh Tuan
                  Jul 11 '17 at 9:14










                • $begingroup$
                  This should be higher IMO. I think to have an intuition (geometrical understanding) of the determinant, you need a geometrical understanding of matrices first.
                  $endgroup$
                  – gwg
                  Nov 1 '18 at 14:03
















                37












                $begingroup$

                Here is a recording of my lecture on the geometric definition of determinants:



                Geometric definition of determinants



                It has elements from the answers by Jamie Banks and John Cook, and goes into details in a leisurely manner.






                share|cite|improve this answer











                $endgroup$









                • 1




                  $begingroup$
                  +1 for a detailed explaination video. You should put the disclosure that you are the lecturer. At first I thought you are a student in the class. By the way, I would like to see you make a full lectures list about linear algebra. It would be very nice.
                  $endgroup$
                  – Anh Tuan
                  Jul 11 '17 at 9:14










                • $begingroup$
                  This should be higher IMO. I think to have an intuition (geometrical understanding) of the determinant, you need a geometrical understanding of matrices first.
                  $endgroup$
                  – gwg
                  Nov 1 '18 at 14:03














                37












                37








                37





                $begingroup$

                Here is a recording of my lecture on the geometric definition of determinants:



                Geometric definition of determinants



                It has elements from the answers by Jamie Banks and John Cook, and goes into details in a leisurely manner.






                share|cite|improve this answer











                $endgroup$



                Here is a recording of my lecture on the geometric definition of determinants:



                Geometric definition of determinants



                It has elements from the answers by Jamie Banks and John Cook, and goes into details in a leisurely manner.







                share|cite|improve this answer














                share|cite|improve this answer



                share|cite|improve this answer








                edited Jul 24 '17 at 6:37

























                answered Dec 18 '12 at 10:46









                Amritanshu PrasadAmritanshu Prasad

                1,0861013




                1,0861013








                • 1




                  $begingroup$
                  +1 for a detailed explaination video. You should put the disclosure that you are the lecturer. At first I thought you are a student in the class. By the way, I would like to see you make a full lectures list about linear algebra. It would be very nice.
                  $endgroup$
                  – Anh Tuan
                  Jul 11 '17 at 9:14










                • $begingroup$
                  This should be higher IMO. I think to have an intuition (geometrical understanding) of the determinant, you need a geometrical understanding of matrices first.
                  $endgroup$
                  – gwg
                  Nov 1 '18 at 14:03














                • 1




                  $begingroup$
                  +1 for a detailed explaination video. You should put the disclosure that you are the lecturer. At first I thought you are a student in the class. By the way, I would like to see you make a full lectures list about linear algebra. It would be very nice.
                  $endgroup$
                  – Anh Tuan
                  Jul 11 '17 at 9:14










                • $begingroup$
                  This should be higher IMO. I think to have an intuition (geometrical understanding) of the determinant, you need a geometrical understanding of matrices first.
                  $endgroup$
                  – gwg
                  Nov 1 '18 at 14:03








                1




                1




                $begingroup$
                +1 for a detailed explaination video. You should put the disclosure that you are the lecturer. At first I thought you are a student in the class. By the way, I would like to see you make a full lectures list about linear algebra. It would be very nice.
                $endgroup$
                – Anh Tuan
                Jul 11 '17 at 9:14




                $begingroup$
                +1 for a detailed explaination video. You should put the disclosure that you are the lecturer. At first I thought you are a student in the class. By the way, I would like to see you make a full lectures list about linear algebra. It would be very nice.
                $endgroup$
                – Anh Tuan
                Jul 11 '17 at 9:14












                $begingroup$
                This should be higher IMO. I think to have an intuition (geometrical understanding) of the determinant, you need a geometrical understanding of matrices first.
                $endgroup$
                – gwg
                Nov 1 '18 at 14:03




                $begingroup$
                This should be higher IMO. I think to have an intuition (geometrical understanding) of the determinant, you need a geometrical understanding of matrices first.
                $endgroup$
                – gwg
                Nov 1 '18 at 14:03











                33












                $begingroup$

                I too find the way determinants are treated in exterior algebra most intuitive. The definition is given on page 46 of Landsberg's "Tensors: Geometry and Applications". Two examples below will tell you everything you need to know.



                Say, you are give a matrix
                $$A=begin{pmatrix}a&b\c&dend{pmatrix}$$
                and asked to compute its determinant. You can think of the matrix as a linear operator $f:mathbb R^2tomathbb R^2$ defined by



                $$begin{pmatrix}x\yend{pmatrix}mapstobegin{pmatrix}a&b\c&dend{pmatrix}begin{pmatrix}x\yend{pmatrix}.$$



                If you define the standard basis vector by $e_1=begin{pmatrix}1\0end{pmatrix}$ and $e_2=begin{pmatrix}0\1end{pmatrix}$, you can then define $f$ by the values it assumes on the basis vectors: $f(e_1)=ae_1+ce_2$ and $f(e_2)=be_1+de_2$.



                The linear operator $f$ is extended to bivectors by
                $$f(e_1wedge e_2)=f(e_1)wedge f(e_2).$$



                Then you can write



                $$f(e_1wedge e_2)=(ae_1+ce_2)wedge(be_1+de_2)=(ad-bc)e_1wedge e_2,$$



                where I used distributivity and anticommutativity of the wedge product (the latter implies $awedge a=0$ for any vector $a$). So, we get the determinant as a scalar factor in the above equation, that is



                $$f(e_1wedge e_2)=det(A),e_1wedge e_2.$$



                The same procedure works for 3-by-3 matrices, you just need to use a trivector. Say, you are given $$B=begin{pmatrix}a_{11}&a_{12}&a_{13}\a_{21}&a_{22}&a_{23}\a_{31}&a_{32}&a_{33}end{pmatrix}.$$



                It defines a linear operator $g:mathbb R^3to mathbb R^3$



                $$begin{pmatrix}x\y\zend{pmatrix}mapsto begin{pmatrix}a_{11}&a_{12}&a_{13}\a_{21}&a_{22}&a_{23}\a_{31}&a_{32}&a_{33}end{pmatrix} begin{pmatrix}x\y\zend{pmatrix},$$



                for which we have



                $$g(e_1)=a_{11}e_1+a_{21}e_2+a_{31}e_3,quad g(e_2)=a_{12}e_1+a_{22}e_2+a_{32}e_3,quad g(e_3)=a_{13}e_1+a_{23}e_2+a_{33}e_3$$



                on the standard basis $e_1=begin{pmatrix}1\0\0end{pmatrix}$, $e_2=begin{pmatrix}0\1\0end{pmatrix}$, $e_3=begin{pmatrix}0\0\1end{pmatrix}$. The operator $g$ is extended to trivectors by
                $$g(e_1wedge e_2wedge e_3)=g(e_1)wedge g(e_2)wedge g(e_3),$$



                which gives



                $$g(e_1wedge e_2wedge e_3)=(a_{11}e_1+a_{21}e_2+a_{31}e_3)wedge(a_{12}e_1+a_{22}e_2+a_{32}e_3)wedge(a_{13}e_1+a_{23}e_2+a_{33}e_3).$$



                If you then follows the rules of $wedge$ such as distributivity, anticommutativity, and associativity, you get
                $$g(e_1wedge e_2wedge e_3)=det(B), e_1wedge e_2wedge e_3.$$



                It works in exactly the same way in higher dimensions.






                share|cite|improve this answer











                $endgroup$









                • 2




                  $begingroup$
                  Beautiful answer. As someone who doesn't know any exterior algebra, this also serves as a motivation for the wedge product.
                  $endgroup$
                  – 6005
                  Nov 15 '16 at 1:37


















                33












                $begingroup$

                I too find the way determinants are treated in exterior algebra most intuitive. The definition is given on page 46 of Landsberg's "Tensors: Geometry and Applications". Two examples below will tell you everything you need to know.



                Say, you are give a matrix
                $$A=begin{pmatrix}a&b\c&dend{pmatrix}$$
                and asked to compute its determinant. You can think of the matrix as a linear operator $f:mathbb R^2tomathbb R^2$ defined by



                $$begin{pmatrix}x\yend{pmatrix}mapstobegin{pmatrix}a&b\c&dend{pmatrix}begin{pmatrix}x\yend{pmatrix}.$$



                If you define the standard basis vector by $e_1=begin{pmatrix}1\0end{pmatrix}$ and $e_2=begin{pmatrix}0\1end{pmatrix}$, you can then define $f$ by the values it assumes on the basis vectors: $f(e_1)=ae_1+ce_2$ and $f(e_2)=be_1+de_2$.



                The linear operator $f$ is extended to bivectors by
                $$f(e_1wedge e_2)=f(e_1)wedge f(e_2).$$



                Then you can write



                $$f(e_1wedge e_2)=(ae_1+ce_2)wedge(be_1+de_2)=(ad-bc)e_1wedge e_2,$$



                where I used distributivity and anticommutativity of the wedge product (the latter implies $awedge a=0$ for any vector $a$). So, we get the determinant as a scalar factor in the above equation, that is



                $$f(e_1wedge e_2)=det(A),e_1wedge e_2.$$



                The same procedure works for 3-by-3 matrices, you just need to use a trivector. Say, you are given $$B=begin{pmatrix}a_{11}&a_{12}&a_{13}\a_{21}&a_{22}&a_{23}\a_{31}&a_{32}&a_{33}end{pmatrix}.$$



                It defines a linear operator $g:mathbb R^3to mathbb R^3$



                $$begin{pmatrix}x\y\zend{pmatrix}mapsto begin{pmatrix}a_{11}&a_{12}&a_{13}\a_{21}&a_{22}&a_{23}\a_{31}&a_{32}&a_{33}end{pmatrix} begin{pmatrix}x\y\zend{pmatrix},$$



                for which we have



                $$g(e_1)=a_{11}e_1+a_{21}e_2+a_{31}e_3,quad g(e_2)=a_{12}e_1+a_{22}e_2+a_{32}e_3,quad g(e_3)=a_{13}e_1+a_{23}e_2+a_{33}e_3$$



                on the standard basis $e_1=begin{pmatrix}1\0\0end{pmatrix}$, $e_2=begin{pmatrix}0\1\0end{pmatrix}$, $e_3=begin{pmatrix}0\0\1end{pmatrix}$. The operator $g$ is extended to trivectors by
                $$g(e_1wedge e_2wedge e_3)=g(e_1)wedge g(e_2)wedge g(e_3),$$



                which gives



                $$g(e_1wedge e_2wedge e_3)=(a_{11}e_1+a_{21}e_2+a_{31}e_3)wedge(a_{12}e_1+a_{22}e_2+a_{32}e_3)wedge(a_{13}e_1+a_{23}e_2+a_{33}e_3).$$



                If you then follows the rules of $wedge$ such as distributivity, anticommutativity, and associativity, you get
                $$g(e_1wedge e_2wedge e_3)=det(B), e_1wedge e_2wedge e_3.$$



                It works in exactly the same way in higher dimensions.






                share|cite|improve this answer











                $endgroup$









                • 2




                  $begingroup$
                  Beautiful answer. As someone who doesn't know any exterior algebra, this also serves as a motivation for the wedge product.
                  $endgroup$
                  – 6005
                  Nov 15 '16 at 1:37
















                33












                33








                33





                $begingroup$

                I too find the way determinants are treated in exterior algebra most intuitive. The definition is given on page 46 of Landsberg's "Tensors: Geometry and Applications". Two examples below will tell you everything you need to know.



                Say, you are give a matrix
                $$A=begin{pmatrix}a&b\c&dend{pmatrix}$$
                and asked to compute its determinant. You can think of the matrix as a linear operator $f:mathbb R^2tomathbb R^2$ defined by



                $$begin{pmatrix}x\yend{pmatrix}mapstobegin{pmatrix}a&b\c&dend{pmatrix}begin{pmatrix}x\yend{pmatrix}.$$



                If you define the standard basis vector by $e_1=begin{pmatrix}1\0end{pmatrix}$ and $e_2=begin{pmatrix}0\1end{pmatrix}$, you can then define $f$ by the values it assumes on the basis vectors: $f(e_1)=ae_1+ce_2$ and $f(e_2)=be_1+de_2$.



                The linear operator $f$ is extended to bivectors by
                $$f(e_1wedge e_2)=f(e_1)wedge f(e_2).$$



                Then you can write



                $$f(e_1wedge e_2)=(ae_1+ce_2)wedge(be_1+de_2)=(ad-bc)e_1wedge e_2,$$



                where I used distributivity and anticommutativity of the wedge product (the latter implies $awedge a=0$ for any vector $a$). So, we get the determinant as a scalar factor in the above equation, that is



                $$f(e_1wedge e_2)=det(A),e_1wedge e_2.$$



                The same procedure works for 3-by-3 matrices, you just need to use a trivector. Say, you are given $$B=begin{pmatrix}a_{11}&a_{12}&a_{13}\a_{21}&a_{22}&a_{23}\a_{31}&a_{32}&a_{33}end{pmatrix}.$$



                It defines a linear operator $g:mathbb R^3to mathbb R^3$



                $$begin{pmatrix}x\y\zend{pmatrix}mapsto begin{pmatrix}a_{11}&a_{12}&a_{13}\a_{21}&a_{22}&a_{23}\a_{31}&a_{32}&a_{33}end{pmatrix} begin{pmatrix}x\y\zend{pmatrix},$$



                for which we have



                $$g(e_1)=a_{11}e_1+a_{21}e_2+a_{31}e_3,quad g(e_2)=a_{12}e_1+a_{22}e_2+a_{32}e_3,quad g(e_3)=a_{13}e_1+a_{23}e_2+a_{33}e_3$$



                on the standard basis $e_1=begin{pmatrix}1\0\0end{pmatrix}$, $e_2=begin{pmatrix}0\1\0end{pmatrix}$, $e_3=begin{pmatrix}0\0\1end{pmatrix}$. The operator $g$ is extended to trivectors by
                $$g(e_1wedge e_2wedge e_3)=g(e_1)wedge g(e_2)wedge g(e_3),$$



                which gives



                $$g(e_1wedge e_2wedge e_3)=(a_{11}e_1+a_{21}e_2+a_{31}e_3)wedge(a_{12}e_1+a_{22}e_2+a_{32}e_3)wedge(a_{13}e_1+a_{23}e_2+a_{33}e_3).$$



                If you then follows the rules of $wedge$ such as distributivity, anticommutativity, and associativity, you get
                $$g(e_1wedge e_2wedge e_3)=det(B), e_1wedge e_2wedge e_3.$$



                It works in exactly the same way in higher dimensions.






                share|cite|improve this answer











                $endgroup$



                I too find the way determinants are treated in exterior algebra most intuitive. The definition is given on page 46 of Landsberg's "Tensors: Geometry and Applications". Two examples below will tell you everything you need to know.



                Say, you are give a matrix
                $$A=begin{pmatrix}a&b\c&dend{pmatrix}$$
                and asked to compute its determinant. You can think of the matrix as a linear operator $f:mathbb R^2tomathbb R^2$ defined by



                $$begin{pmatrix}x\yend{pmatrix}mapstobegin{pmatrix}a&b\c&dend{pmatrix}begin{pmatrix}x\yend{pmatrix}.$$



                If you define the standard basis vector by $e_1=begin{pmatrix}1\0end{pmatrix}$ and $e_2=begin{pmatrix}0\1end{pmatrix}$, you can then define $f$ by the values it assumes on the basis vectors: $f(e_1)=ae_1+ce_2$ and $f(e_2)=be_1+de_2$.



                The linear operator $f$ is extended to bivectors by
                $$f(e_1wedge e_2)=f(e_1)wedge f(e_2).$$



                Then you can write



                $$f(e_1wedge e_2)=(ae_1+ce_2)wedge(be_1+de_2)=(ad-bc)e_1wedge e_2,$$



                where I used distributivity and anticommutativity of the wedge product (the latter implies $awedge a=0$ for any vector $a$). So, we get the determinant as a scalar factor in the above equation, that is



                $$f(e_1wedge e_2)=det(A),e_1wedge e_2.$$



                The same procedure works for 3-by-3 matrices, you just need to use a trivector. Say, you are given $$B=begin{pmatrix}a_{11}&a_{12}&a_{13}\a_{21}&a_{22}&a_{23}\a_{31}&a_{32}&a_{33}end{pmatrix}.$$



                It defines a linear operator $g:mathbb R^3to mathbb R^3$



                $$begin{pmatrix}x\y\zend{pmatrix}mapsto begin{pmatrix}a_{11}&a_{12}&a_{13}\a_{21}&a_{22}&a_{23}\a_{31}&a_{32}&a_{33}end{pmatrix} begin{pmatrix}x\y\zend{pmatrix},$$



                for which we have



                $$g(e_1)=a_{11}e_1+a_{21}e_2+a_{31}e_3,quad g(e_2)=a_{12}e_1+a_{22}e_2+a_{32}e_3,quad g(e_3)=a_{13}e_1+a_{23}e_2+a_{33}e_3$$



                on the standard basis $e_1=begin{pmatrix}1\0\0end{pmatrix}$, $e_2=begin{pmatrix}0\1\0end{pmatrix}$, $e_3=begin{pmatrix}0\0\1end{pmatrix}$. The operator $g$ is extended to trivectors by
                $$g(e_1wedge e_2wedge e_3)=g(e_1)wedge g(e_2)wedge g(e_3),$$



                which gives



                $$g(e_1wedge e_2wedge e_3)=(a_{11}e_1+a_{21}e_2+a_{31}e_3)wedge(a_{12}e_1+a_{22}e_2+a_{32}e_3)wedge(a_{13}e_1+a_{23}e_2+a_{33}e_3).$$



                If you then follows the rules of $wedge$ such as distributivity, anticommutativity, and associativity, you get
                $$g(e_1wedge e_2wedge e_3)=det(B), e_1wedge e_2wedge e_3.$$



                It works in exactly the same way in higher dimensions.







                share|cite|improve this answer














                share|cite|improve this answer



                share|cite|improve this answer








                edited Nov 11 '13 at 4:23

























                answered Nov 11 '13 at 4:17









                Andrey SokolovAndrey Sokolov

                1,006813




                1,006813








                • 2




                  $begingroup$
                  Beautiful answer. As someone who doesn't know any exterior algebra, this also serves as a motivation for the wedge product.
                  $endgroup$
                  – 6005
                  Nov 15 '16 at 1:37
















                • 2




                  $begingroup$
                  Beautiful answer. As someone who doesn't know any exterior algebra, this also serves as a motivation for the wedge product.
                  $endgroup$
                  – 6005
                  Nov 15 '16 at 1:37










                2




                2




                $begingroup$
                Beautiful answer. As someone who doesn't know any exterior algebra, this also serves as a motivation for the wedge product.
                $endgroup$
                – 6005
                Nov 15 '16 at 1:37






                $begingroup$
                Beautiful answer. As someone who doesn't know any exterior algebra, this also serves as a motivation for the wedge product.
                $endgroup$
                – 6005
                Nov 15 '16 at 1:37













                27












                $begingroup$

                The top exterior power of an $n$-dimensional vector space $V$ is one-dimensional. Its elements are sometimes called pseudoscalars, and they represent oriented $n$-dimensional volume elements.



                A linear operator $f$ on $V$ can be extended to a linear map on the exterior algebra according to the rules $f(alpha) = alpha$ for $alpha$ a scalar and $f(A wedge B) = f(A) wedge f(B), f(A + B) = f(A) + f(B)$ for $A$ and $B$ blades of arbitrary grade. Trivia: some authors call this extension an outermorphism. The extended map will be grade-preserving; that is, if $A$ is a homogeneous element of the exterior algebra of grade $m$, then $f(A)$ will also have grade $m$. (This can be verified from the properties of the extended map I just listed.)



                All this implies that a linear map on the exterior algebra of $V$ once restricted to the top exterior power reduces to multiplication by a constant: the determinant of the original linear transformation. Since pseudoscalars represent oriented volume elements, this means that the determinant is precisely the factor by which the map scales oriented volumes.






                share|cite|improve this answer











                $endgroup$









                • 9




                  $begingroup$
                  It's worth mentioning here that, as abstract as the construction by the top exterior power is, it's the cleanest way to derive the permutation formula.
                  $endgroup$
                  – Qiaochu Yuan
                  Jul 31 '10 at 8:14










                • $begingroup$
                  I give a couple of examples of how this works in my answer.
                  $endgroup$
                  – Andrey Sokolov
                  Nov 11 '13 at 4:26
















                27












                $begingroup$

                The top exterior power of an $n$-dimensional vector space $V$ is one-dimensional. Its elements are sometimes called pseudoscalars, and they represent oriented $n$-dimensional volume elements.



                A linear operator $f$ on $V$ can be extended to a linear map on the exterior algebra according to the rules $f(alpha) = alpha$ for $alpha$ a scalar and $f(A wedge B) = f(A) wedge f(B), f(A + B) = f(A) + f(B)$ for $A$ and $B$ blades of arbitrary grade. Trivia: some authors call this extension an outermorphism. The extended map will be grade-preserving; that is, if $A$ is a homogeneous element of the exterior algebra of grade $m$, then $f(A)$ will also have grade $m$. (This can be verified from the properties of the extended map I just listed.)



                All this implies that a linear map on the exterior algebra of $V$ once restricted to the top exterior power reduces to multiplication by a constant: the determinant of the original linear transformation. Since pseudoscalars represent oriented volume elements, this means that the determinant is precisely the factor by which the map scales oriented volumes.






                share|cite|improve this answer











                $endgroup$









                • 9




                  $begingroup$
                  It's worth mentioning here that, as abstract as the construction by the top exterior power is, it's the cleanest way to derive the permutation formula.
                  $endgroup$
                  – Qiaochu Yuan
                  Jul 31 '10 at 8:14










                • $begingroup$
                  I give a couple of examples of how this works in my answer.
                  $endgroup$
                  – Andrey Sokolov
                  Nov 11 '13 at 4:26














                27












                27








                27





                $begingroup$

                The top exterior power of an $n$-dimensional vector space $V$ is one-dimensional. Its elements are sometimes called pseudoscalars, and they represent oriented $n$-dimensional volume elements.



                A linear operator $f$ on $V$ can be extended to a linear map on the exterior algebra according to the rules $f(alpha) = alpha$ for $alpha$ a scalar and $f(A wedge B) = f(A) wedge f(B), f(A + B) = f(A) + f(B)$ for $A$ and $B$ blades of arbitrary grade. Trivia: some authors call this extension an outermorphism. The extended map will be grade-preserving; that is, if $A$ is a homogeneous element of the exterior algebra of grade $m$, then $f(A)$ will also have grade $m$. (This can be verified from the properties of the extended map I just listed.)



                All this implies that a linear map on the exterior algebra of $V$ once restricted to the top exterior power reduces to multiplication by a constant: the determinant of the original linear transformation. Since pseudoscalars represent oriented volume elements, this means that the determinant is precisely the factor by which the map scales oriented volumes.






                share|cite|improve this answer











                $endgroup$



                The top exterior power of an $n$-dimensional vector space $V$ is one-dimensional. Its elements are sometimes called pseudoscalars, and they represent oriented $n$-dimensional volume elements.



                A linear operator $f$ on $V$ can be extended to a linear map on the exterior algebra according to the rules $f(alpha) = alpha$ for $alpha$ a scalar and $f(A wedge B) = f(A) wedge f(B), f(A + B) = f(A) + f(B)$ for $A$ and $B$ blades of arbitrary grade. Trivia: some authors call this extension an outermorphism. The extended map will be grade-preserving; that is, if $A$ is a homogeneous element of the exterior algebra of grade $m$, then $f(A)$ will also have grade $m$. (This can be verified from the properties of the extended map I just listed.)



                All this implies that a linear map on the exterior algebra of $V$ once restricted to the top exterior power reduces to multiplication by a constant: the determinant of the original linear transformation. Since pseudoscalars represent oriented volume elements, this means that the determinant is precisely the factor by which the map scales oriented volumes.







                share|cite|improve this answer














                share|cite|improve this answer



                share|cite|improve this answer








                edited Jul 29 '10 at 3:16

























                answered Jul 29 '10 at 0:36









                Zach ConnZach Conn

                2,85011629




                2,85011629








                • 9




                  $begingroup$
                  It's worth mentioning here that, as abstract as the construction by the top exterior power is, it's the cleanest way to derive the permutation formula.
                  $endgroup$
                  – Qiaochu Yuan
                  Jul 31 '10 at 8:14










                • $begingroup$
                  I give a couple of examples of how this works in my answer.
                  $endgroup$
                  – Andrey Sokolov
                  Nov 11 '13 at 4:26














                • 9




                  $begingroup$
                  It's worth mentioning here that, as abstract as the construction by the top exterior power is, it's the cleanest way to derive the permutation formula.
                  $endgroup$
                  – Qiaochu Yuan
                  Jul 31 '10 at 8:14










                • $begingroup$
                  I give a couple of examples of how this works in my answer.
                  $endgroup$
                  – Andrey Sokolov
                  Nov 11 '13 at 4:26








                9




                9




                $begingroup$
                It's worth mentioning here that, as abstract as the construction by the top exterior power is, it's the cleanest way to derive the permutation formula.
                $endgroup$
                – Qiaochu Yuan
                Jul 31 '10 at 8:14




                $begingroup$
                It's worth mentioning here that, as abstract as the construction by the top exterior power is, it's the cleanest way to derive the permutation formula.
                $endgroup$
                – Qiaochu Yuan
                Jul 31 '10 at 8:14












                $begingroup$
                I give a couple of examples of how this works in my answer.
                $endgroup$
                – Andrey Sokolov
                Nov 11 '13 at 4:26




                $begingroup$
                I give a couple of examples of how this works in my answer.
                $endgroup$
                – Andrey Sokolov
                Nov 11 '13 at 4:26











                26












                $begingroup$

                For the record I'll try to give a reply to this old question, since I think some elements can be added to what has been already said.



                Even though they are basically just (complicated) expressions, determinants can be mysterious when first encountered. Questions that arise naturally are: (1) how are they defined in general?, (2) what are their important properties?, (3) why do they exist?, (4) why should we care?, and (5) why does their expression get so huge for large matrices?



                Since $2times2$ and $3times3$ determinants are easily defined explicitly, question (1) can wait. While (2) has many answers, the most important ones are, to me: determinants detect (by becoming 0) the linear dependence of $n$ vectors in dimension $n$, and they are an expression in the coordinates of those vectors (rather than for instance an algorithm). If you have a family of vectors that depend (or at least one of them depends) on a parameter, and you need to know for which parameter values they are linearly dependent, than trying to use for instance Gaussian elimination to detect linear dependence can run into trouble: one might need assumptions on the parameter to assure some coefficient is nonzero, and even then dividing by it gives very messy expressions. Provided the number of vectors equals the dimension $n$ of the space, taking a determinant will however immediately transform the question into an equation for the parameter (which one may or may not be capable of solving, but that is another matter). This is exactly how one obtains an equation in eigenvalue problems, in case you've seen those. This provides a first answer to (4). (But there is a lot more you can do with determinants once you get used to them.)



                As for question (3), the mystery of why determinants exist in the first place can be reduced by considering the situation where one has $n-1$ given linearly independent vectors, and asks when a final unknown vector $vec x$ will remain independent from them, in terms of its coordinates. The answer is that it usually will, in fact always unless $vec x$ happens to be in the linear span $S$ of those $n-1$ vectors, which is a subspace of dimension $n-1$. For instance, if $n=2$ (with one vector $vec v$ given) the answer is "unless $vec x$ is a scalar multiple of $vec v$". Now if one imagines a fixed (nonzero) linear combination of the coordinates of $vec x$ (the technical term is a linear form on the space), then it will become $0$ precisely when $vec x$ is in some subspace of dimension $n-1$. With some luck, this can be arranged to be precisely the linear span $S$. (In fact no luck is involved: if one extends the $n-1$ vectors by one more vector to a basis, then expressing $vec x$ in that basis and taking its final coordinate will define such a linear form; however you can ignore this argument unless you are particularly suspicious.) Now the crucial observation is that not only does such a linear combination exist, its coefficients can be taken to be expressions in the coordinates of our $n-1$ vectors. For instance in the case $n=2$ if one puts $vec v={achoose b}$ and $vec x={x_1choose x_2}$, then the linear combination $-bx_1+ax_2$ does the job (it becomes 0 precisely when $vec x$ is a scalar multiple of $vec v$), and $-b$ and $a$ are clearly expressions in the coordinates of $vec v$. In fact they are linear expressions. For $n=3$ with two given vectors, the expressions for the coefficients of the linear combination are more complicated, but they can still be explicitly written down (each coefficient is the difference of two products of coordinates, one form each vector). These expressions are linear in each of the vectors, if the other one is fixed.



                Thus one arrives at the notion of a multilinear expression (or form). The determinant is in fact a multilinear form: an expression that depends on $n$ vectors, and is linear in each of them taken individually (fixing the other vectors to arbitrary values). This means it is a sum of terms, each of which is the product of a coefficient, and of one coordinate each of all the $n$ vectors. But even ignoring the coefficients, there are many such terms possible: a whopping $n^n$ of them!



                However, we want an expression that becomes $0$ when the vectors are linearly dependent. Now the magic (sort of) is that even the seemingly much weaker requirement that the expression becomes $0$ when two successive vectors among the $n$ are equal will assure this, and it will moreover almost force the form of our expression upon us. Multilinear forms that satisfy this requirement are called alternating. I'll skip the (easy) arguments, but an alternating form cannot involve terms that take the same coordinate of any two different vectors, and they must change sign whenever one interchanges the role of two vectors (in particular they cannot be symmetric with respect to the vectors, even though the notion of linear dependence is symmetric; note that already $-bx_1+ax_2$ is not symmetric with respect to interchange of $(a,b)$ and $(x_1,x_2)$). Thus any one term must involve each of the $n$ coordinates once, but not necessarily in order: it applies a permutation of the coordinates $1,2,ldots,n$ to the successive vectors. Moreover, if a term involves one such permutation, then any term obtained by interchanging two positions in the permutation must also occur, with an opposite coefficient. But any two permutations can be transformed into one another by repeatedly interchanging two positions; so if there are any terms at all, then there must be terms for all $n!$ permutations, and their coefficients are all equal or opposite. This explains question (5), why the determinant is such a huge expression when $n$ is large.



                Finally the fact that determinants exist turns out to be directly related to the fact that signs can be associated to all permutations in such a way that interchanging entries always changes the sign, which is part of the answer to question (3).
                As for question (1), we can now say that the determinant is uniquely determined by being an $n$-linear alternating expression in the entries of $n$ column vectors, which contains a term consisting of the product of their coordinates $1,2,ldots,n$ in that order (the diagonal term) with coefficient $+1$. The explicit expression is a sum over all $n!$ permutations, the corresponding term being obtained by applying those coordinates in permuted order, and with the sign of the permutation as coefficient. A lot more can be said about question (2), but I'll stop here.






                share|cite|improve this answer











                $endgroup$


















                  26












                  $begingroup$

                  For the record I'll try to give a reply to this old question, since I think some elements can be added to what has been already said.



                  Even though they are basically just (complicated) expressions, determinants can be mysterious when first encountered. Questions that arise naturally are: (1) how are they defined in general?, (2) what are their important properties?, (3) why do they exist?, (4) why should we care?, and (5) why does their expression get so huge for large matrices?



                  Since $2times2$ and $3times3$ determinants are easily defined explicitly, question (1) can wait. While (2) has many answers, the most important ones are, to me: determinants detect (by becoming 0) the linear dependence of $n$ vectors in dimension $n$, and they are an expression in the coordinates of those vectors (rather than for instance an algorithm). If you have a family of vectors that depend (or at least one of them depends) on a parameter, and you need to know for which parameter values they are linearly dependent, than trying to use for instance Gaussian elimination to detect linear dependence can run into trouble: one might need assumptions on the parameter to assure some coefficient is nonzero, and even then dividing by it gives very messy expressions. Provided the number of vectors equals the dimension $n$ of the space, taking a determinant will however immediately transform the question into an equation for the parameter (which one may or may not be capable of solving, but that is another matter). This is exactly how one obtains an equation in eigenvalue problems, in case you've seen those. This provides a first answer to (4). (But there is a lot more you can do with determinants once you get used to them.)



                  As for question (3), the mystery of why determinants exist in the first place can be reduced by considering the situation where one has $n-1$ given linearly independent vectors, and asks when a final unknown vector $vec x$ will remain independent from them, in terms of its coordinates. The answer is that it usually will, in fact always unless $vec x$ happens to be in the linear span $S$ of those $n-1$ vectors, which is a subspace of dimension $n-1$. For instance, if $n=2$ (with one vector $vec v$ given) the answer is "unless $vec x$ is a scalar multiple of $vec v$". Now if one imagines a fixed (nonzero) linear combination of the coordinates of $vec x$ (the technical term is a linear form on the space), then it will become $0$ precisely when $vec x$ is in some subspace of dimension $n-1$. With some luck, this can be arranged to be precisely the linear span $S$. (In fact no luck is involved: if one extends the $n-1$ vectors by one more vector to a basis, then expressing $vec x$ in that basis and taking its final coordinate will define such a linear form; however you can ignore this argument unless you are particularly suspicious.) Now the crucial observation is that not only does such a linear combination exist, its coefficients can be taken to be expressions in the coordinates of our $n-1$ vectors. For instance in the case $n=2$ if one puts $vec v={achoose b}$ and $vec x={x_1choose x_2}$, then the linear combination $-bx_1+ax_2$ does the job (it becomes 0 precisely when $vec x$ is a scalar multiple of $vec v$), and $-b$ and $a$ are clearly expressions in the coordinates of $vec v$. In fact they are linear expressions. For $n=3$ with two given vectors, the expressions for the coefficients of the linear combination are more complicated, but they can still be explicitly written down (each coefficient is the difference of two products of coordinates, one form each vector). These expressions are linear in each of the vectors, if the other one is fixed.



                  Thus one arrives at the notion of a multilinear expression (or form). The determinant is in fact a multilinear form: an expression that depends on $n$ vectors, and is linear in each of them taken individually (fixing the other vectors to arbitrary values). This means it is a sum of terms, each of which is the product of a coefficient, and of one coordinate each of all the $n$ vectors. But even ignoring the coefficients, there are many such terms possible: a whopping $n^n$ of them!



                  However, we want an expression that becomes $0$ when the vectors are linearly dependent. Now the magic (sort of) is that even the seemingly much weaker requirement that the expression becomes $0$ when two successive vectors among the $n$ are equal will assure this, and it will moreover almost force the form of our expression upon us. Multilinear forms that satisfy this requirement are called alternating. I'll skip the (easy) arguments, but an alternating form cannot involve terms that take the same coordinate of any two different vectors, and they must change sign whenever one interchanges the role of two vectors (in particular they cannot be symmetric with respect to the vectors, even though the notion of linear dependence is symmetric; note that already $-bx_1+ax_2$ is not symmetric with respect to interchange of $(a,b)$ and $(x_1,x_2)$). Thus any one term must involve each of the $n$ coordinates once, but not necessarily in order: it applies a permutation of the coordinates $1,2,ldots,n$ to the successive vectors. Moreover, if a term involves one such permutation, then any term obtained by interchanging two positions in the permutation must also occur, with an opposite coefficient. But any two permutations can be transformed into one another by repeatedly interchanging two positions; so if there are any terms at all, then there must be terms for all $n!$ permutations, and their coefficients are all equal or opposite. This explains question (5), why the determinant is such a huge expression when $n$ is large.



                  Finally the fact that determinants exist turns out to be directly related to the fact that signs can be associated to all permutations in such a way that interchanging entries always changes the sign, which is part of the answer to question (3).
                  As for question (1), we can now say that the determinant is uniquely determined by being an $n$-linear alternating expression in the entries of $n$ column vectors, which contains a term consisting of the product of their coordinates $1,2,ldots,n$ in that order (the diagonal term) with coefficient $+1$. The explicit expression is a sum over all $n!$ permutations, the corresponding term being obtained by applying those coordinates in permuted order, and with the sign of the permutation as coefficient. A lot more can be said about question (2), but I'll stop here.






                  share|cite|improve this answer











                  $endgroup$
















                    26












                    26








                    26





                    $begingroup$

                    For the record I'll try to give a reply to this old question, since I think some elements can be added to what has been already said.



                    Even though they are basically just (complicated) expressions, determinants can be mysterious when first encountered. Questions that arise naturally are: (1) how are they defined in general?, (2) what are their important properties?, (3) why do they exist?, (4) why should we care?, and (5) why does their expression get so huge for large matrices?



                    Since $2times2$ and $3times3$ determinants are easily defined explicitly, question (1) can wait. While (2) has many answers, the most important ones are, to me: determinants detect (by becoming 0) the linear dependence of $n$ vectors in dimension $n$, and they are an expression in the coordinates of those vectors (rather than for instance an algorithm). If you have a family of vectors that depend (or at least one of them depends) on a parameter, and you need to know for which parameter values they are linearly dependent, than trying to use for instance Gaussian elimination to detect linear dependence can run into trouble: one might need assumptions on the parameter to assure some coefficient is nonzero, and even then dividing by it gives very messy expressions. Provided the number of vectors equals the dimension $n$ of the space, taking a determinant will however immediately transform the question into an equation for the parameter (which one may or may not be capable of solving, but that is another matter). This is exactly how one obtains an equation in eigenvalue problems, in case you've seen those. This provides a first answer to (4). (But there is a lot more you can do with determinants once you get used to them.)



                    As for question (3), the mystery of why determinants exist in the first place can be reduced by considering the situation where one has $n-1$ given linearly independent vectors, and asks when a final unknown vector $vec x$ will remain independent from them, in terms of its coordinates. The answer is that it usually will, in fact always unless $vec x$ happens to be in the linear span $S$ of those $n-1$ vectors, which is a subspace of dimension $n-1$. For instance, if $n=2$ (with one vector $vec v$ given) the answer is "unless $vec x$ is a scalar multiple of $vec v$". Now if one imagines a fixed (nonzero) linear combination of the coordinates of $vec x$ (the technical term is a linear form on the space), then it will become $0$ precisely when $vec x$ is in some subspace of dimension $n-1$. With some luck, this can be arranged to be precisely the linear span $S$. (In fact no luck is involved: if one extends the $n-1$ vectors by one more vector to a basis, then expressing $vec x$ in that basis and taking its final coordinate will define such a linear form; however you can ignore this argument unless you are particularly suspicious.) Now the crucial observation is that not only does such a linear combination exist, its coefficients can be taken to be expressions in the coordinates of our $n-1$ vectors. For instance in the case $n=2$ if one puts $vec v={achoose b}$ and $vec x={x_1choose x_2}$, then the linear combination $-bx_1+ax_2$ does the job (it becomes 0 precisely when $vec x$ is a scalar multiple of $vec v$), and $-b$ and $a$ are clearly expressions in the coordinates of $vec v$. In fact they are linear expressions. For $n=3$ with two given vectors, the expressions for the coefficients of the linear combination are more complicated, but they can still be explicitly written down (each coefficient is the difference of two products of coordinates, one form each vector). These expressions are linear in each of the vectors, if the other one is fixed.



                    Thus one arrives at the notion of a multilinear expression (or form). The determinant is in fact a multilinear form: an expression that depends on $n$ vectors, and is linear in each of them taken individually (fixing the other vectors to arbitrary values). This means it is a sum of terms, each of which is the product of a coefficient, and of one coordinate each of all the $n$ vectors. But even ignoring the coefficients, there are many such terms possible: a whopping $n^n$ of them!



                    However, we want an expression that becomes $0$ when the vectors are linearly dependent. Now the magic (sort of) is that even the seemingly much weaker requirement that the expression becomes $0$ when two successive vectors among the $n$ are equal will assure this, and it will moreover almost force the form of our expression upon us. Multilinear forms that satisfy this requirement are called alternating. I'll skip the (easy) arguments, but an alternating form cannot involve terms that take the same coordinate of any two different vectors, and they must change sign whenever one interchanges the role of two vectors (in particular they cannot be symmetric with respect to the vectors, even though the notion of linear dependence is symmetric; note that already $-bx_1+ax_2$ is not symmetric with respect to interchange of $(a,b)$ and $(x_1,x_2)$). Thus any one term must involve each of the $n$ coordinates once, but not necessarily in order: it applies a permutation of the coordinates $1,2,ldots,n$ to the successive vectors. Moreover, if a term involves one such permutation, then any term obtained by interchanging two positions in the permutation must also occur, with an opposite coefficient. But any two permutations can be transformed into one another by repeatedly interchanging two positions; so if there are any terms at all, then there must be terms for all $n!$ permutations, and their coefficients are all equal or opposite. This explains question (5), why the determinant is such a huge expression when $n$ is large.



                    Finally the fact that determinants exist turns out to be directly related to the fact that signs can be associated to all permutations in such a way that interchanging entries always changes the sign, which is part of the answer to question (3).
                    As for question (1), we can now say that the determinant is uniquely determined by being an $n$-linear alternating expression in the entries of $n$ column vectors, which contains a term consisting of the product of their coordinates $1,2,ldots,n$ in that order (the diagonal term) with coefficient $+1$. The explicit expression is a sum over all $n!$ permutations, the corresponding term being obtained by applying those coordinates in permuted order, and with the sign of the permutation as coefficient. A lot more can be said about question (2), but I'll stop here.






                    share|cite|improve this answer











                    $endgroup$



                    For the record I'll try to give a reply to this old question, since I think some elements can be added to what has been already said.



                    Even though they are basically just (complicated) expressions, determinants can be mysterious when first encountered. Questions that arise naturally are: (1) how are they defined in general?, (2) what are their important properties?, (3) why do they exist?, (4) why should we care?, and (5) why does their expression get so huge for large matrices?



                    Since $2times2$ and $3times3$ determinants are easily defined explicitly, question (1) can wait. While (2) has many answers, the most important ones are, to me: determinants detect (by becoming 0) the linear dependence of $n$ vectors in dimension $n$, and they are an expression in the coordinates of those vectors (rather than for instance an algorithm). If you have a family of vectors that depend (or at least one of them depends) on a parameter, and you need to know for which parameter values they are linearly dependent, than trying to use for instance Gaussian elimination to detect linear dependence can run into trouble: one might need assumptions on the parameter to assure some coefficient is nonzero, and even then dividing by it gives very messy expressions. Provided the number of vectors equals the dimension $n$ of the space, taking a determinant will however immediately transform the question into an equation for the parameter (which one may or may not be capable of solving, but that is another matter). This is exactly how one obtains an equation in eigenvalue problems, in case you've seen those. This provides a first answer to (4). (But there is a lot more you can do with determinants once you get used to them.)



                    As for question (3), the mystery of why determinants exist in the first place can be reduced by considering the situation where one has $n-1$ given linearly independent vectors, and asks when a final unknown vector $vec x$ will remain independent from them, in terms of its coordinates. The answer is that it usually will, in fact always unless $vec x$ happens to be in the linear span $S$ of those $n-1$ vectors, which is a subspace of dimension $n-1$. For instance, if $n=2$ (with one vector $vec v$ given) the answer is "unless $vec x$ is a scalar multiple of $vec v$". Now if one imagines a fixed (nonzero) linear combination of the coordinates of $vec x$ (the technical term is a linear form on the space), then it will become $0$ precisely when $vec x$ is in some subspace of dimension $n-1$. With some luck, this can be arranged to be precisely the linear span $S$. (In fact no luck is involved: if one extends the $n-1$ vectors by one more vector to a basis, then expressing $vec x$ in that basis and taking its final coordinate will define such a linear form; however you can ignore this argument unless you are particularly suspicious.) Now the crucial observation is that not only does such a linear combination exist, its coefficients can be taken to be expressions in the coordinates of our $n-1$ vectors. For instance in the case $n=2$ if one puts $vec v={achoose b}$ and $vec x={x_1choose x_2}$, then the linear combination $-bx_1+ax_2$ does the job (it becomes 0 precisely when $vec x$ is a scalar multiple of $vec v$), and $-b$ and $a$ are clearly expressions in the coordinates of $vec v$. In fact they are linear expressions. For $n=3$ with two given vectors, the expressions for the coefficients of the linear combination are more complicated, but they can still be explicitly written down (each coefficient is the difference of two products of coordinates, one form each vector). These expressions are linear in each of the vectors, if the other one is fixed.



                    Thus one arrives at the notion of a multilinear expression (or form). The determinant is in fact a multilinear form: an expression that depends on $n$ vectors, and is linear in each of them taken individually (fixing the other vectors to arbitrary values). This means it is a sum of terms, each of which is the product of a coefficient, and of one coordinate each of all the $n$ vectors. But even ignoring the coefficients, there are many such terms possible: a whopping $n^n$ of them!



                    However, we want an expression that becomes $0$ when the vectors are linearly dependent. Now the magic (sort of) is that even the seemingly much weaker requirement that the expression becomes $0$ when two successive vectors among the $n$ are equal will assure this, and it will moreover almost force the form of our expression upon us. Multilinear forms that satisfy this requirement are called alternating. I'll skip the (easy) arguments, but an alternating form cannot involve terms that take the same coordinate of any two different vectors, and they must change sign whenever one interchanges the role of two vectors (in particular they cannot be symmetric with respect to the vectors, even though the notion of linear dependence is symmetric; note that already $-bx_1+ax_2$ is not symmetric with respect to interchange of $(a,b)$ and $(x_1,x_2)$). Thus any one term must involve each of the $n$ coordinates once, but not necessarily in order: it applies a permutation of the coordinates $1,2,ldots,n$ to the successive vectors. Moreover, if a term involves one such permutation, then any term obtained by interchanging two positions in the permutation must also occur, with an opposite coefficient. But any two permutations can be transformed into one another by repeatedly interchanging two positions; so if there are any terms at all, then there must be terms for all $n!$ permutations, and their coefficients are all equal or opposite. This explains question (5), why the determinant is such a huge expression when $n$ is large.



                    Finally the fact that determinants exist turns out to be directly related to the fact that signs can be associated to all permutations in such a way that interchanging entries always changes the sign, which is part of the answer to question (3).
                    As for question (1), we can now say that the determinant is uniquely determined by being an $n$-linear alternating expression in the entries of $n$ column vectors, which contains a term consisting of the product of their coordinates $1,2,ldots,n$ in that order (the diagonal term) with coefficient $+1$. The explicit expression is a sum over all $n!$ permutations, the corresponding term being obtained by applying those coordinates in permuted order, and with the sign of the permutation as coefficient. A lot more can be said about question (2), but I'll stop here.







                    share|cite|improve this answer














                    share|cite|improve this answer



                    share|cite|improve this answer








                    edited Jun 3 '16 at 5:04

























                    answered Nov 14 '11 at 13:25









                    Marc van LeeuwenMarc van Leeuwen

                    88.3k5111228




                    88.3k5111228























                        12












                        $begingroup$

                        If you have a matrix




                        • $H$
                          then you can calculate the correlationmatrix with

                        • $G = H times H^H$
                          (H^H denotes the complex conjugated and transposed version of $H$).


                        If you do a eigenvalue decomposition of $G$ you get eigenvalues $lambda$ and eigenvectors $v$, that in combination $lambdatimes v$ describes the same space.



                        Now there is the following equation, saying:




                        • Determinant($H*H^H$) = Product of all eigenvalues $lambda$


                        I.e., if you have a $3times3$ matrix $H$ then $G$ is $3times3$ too giving us three eigenvalues.
                        The product of these eigenvalues give as the volume of a cuboid.
                        With every extra dimension/eigenvalue the cuboid gets an extra dimension.






                        share|cite|improve this answer











                        $endgroup$


















                          12












                          $begingroup$

                          If you have a matrix




                          • $H$
                            then you can calculate the correlationmatrix with

                          • $G = H times H^H$
                            (H^H denotes the complex conjugated and transposed version of $H$).


                          If you do a eigenvalue decomposition of $G$ you get eigenvalues $lambda$ and eigenvectors $v$, that in combination $lambdatimes v$ describes the same space.



                          Now there is the following equation, saying:




                          • Determinant($H*H^H$) = Product of all eigenvalues $lambda$


                          I.e., if you have a $3times3$ matrix $H$ then $G$ is $3times3$ too giving us three eigenvalues.
                          The product of these eigenvalues give as the volume of a cuboid.
                          With every extra dimension/eigenvalue the cuboid gets an extra dimension.






                          share|cite|improve this answer











                          $endgroup$
















                            12












                            12








                            12





                            $begingroup$

                            If you have a matrix




                            • $H$
                              then you can calculate the correlationmatrix with

                            • $G = H times H^H$
                              (H^H denotes the complex conjugated and transposed version of $H$).


                            If you do a eigenvalue decomposition of $G$ you get eigenvalues $lambda$ and eigenvectors $v$, that in combination $lambdatimes v$ describes the same space.



                            Now there is the following equation, saying:




                            • Determinant($H*H^H$) = Product of all eigenvalues $lambda$


                            I.e., if you have a $3times3$ matrix $H$ then $G$ is $3times3$ too giving us three eigenvalues.
                            The product of these eigenvalues give as the volume of a cuboid.
                            With every extra dimension/eigenvalue the cuboid gets an extra dimension.






                            share|cite|improve this answer











                            $endgroup$



                            If you have a matrix




                            • $H$
                              then you can calculate the correlationmatrix with

                            • $G = H times H^H$
                              (H^H denotes the complex conjugated and transposed version of $H$).


                            If you do a eigenvalue decomposition of $G$ you get eigenvalues $lambda$ and eigenvectors $v$, that in combination $lambdatimes v$ describes the same space.



                            Now there is the following equation, saying:




                            • Determinant($H*H^H$) = Product of all eigenvalues $lambda$


                            I.e., if you have a $3times3$ matrix $H$ then $G$ is $3times3$ too giving us three eigenvalues.
                            The product of these eigenvalues give as the volume of a cuboid.
                            With every extra dimension/eigenvalue the cuboid gets an extra dimension.







                            share|cite|improve this answer














                            share|cite|improve this answer



                            share|cite|improve this answer








                            edited Nov 11 '11 at 17:31









                            Marc van Leeuwen

                            88.3k5111228




                            88.3k5111228










                            answered Jan 15 '11 at 21:09









                            xerocxeroc

                            24524




                            24524























                                12












                                $begingroup$

                                There are excellent answers here that are very detailed.



                                Here I provide a simpler answer, also discussed in wikipedia. Think of the determinant as the area (in 2D; in 3D it would be the volume, etc.) of the parallelogram made by the vectors:



                                Parallelogram from vectors



                                Keep in mind that the area of a parallelogram is the base $times$ height. Doing some tricks with the dot product, this yields the determinant:



                                $$
                                begin{vmatrix}
                                a & b \
                                c & d
                                end{vmatrix}
                                = ad - bc = Area_{parallelogram}
                                $$



                                You can place the unit vectors for each dimension to test the identity matrix by seeing that:
                                $$
                                begin{vmatrix}
                                1 & 0 \
                                0 & 1
                                end{vmatrix}
                                = ad - bc = 1 times 1 - 0 times 0 = 1 $$



                                This is a volume in more than a 2 by 2 matrix and will be equal to 1 in all cases as the off diagonal elements remove any effect from the only value contributing to the volume as the diagonal product of 1s. It is understood in some contexts that the coordinate system is unmodified.






                                share|cite|improve this answer











                                $endgroup$


















                                  12












                                  $begingroup$

                                  There are excellent answers here that are very detailed.



                                  Here I provide a simpler answer, also discussed in wikipedia. Think of the determinant as the area (in 2D; in 3D it would be the volume, etc.) of the parallelogram made by the vectors:



                                  Parallelogram from vectors



                                  Keep in mind that the area of a parallelogram is the base $times$ height. Doing some tricks with the dot product, this yields the determinant:



                                  $$
                                  begin{vmatrix}
                                  a & b \
                                  c & d
                                  end{vmatrix}
                                  = ad - bc = Area_{parallelogram}
                                  $$



                                  You can place the unit vectors for each dimension to test the identity matrix by seeing that:
                                  $$
                                  begin{vmatrix}
                                  1 & 0 \
                                  0 & 1
                                  end{vmatrix}
                                  = ad - bc = 1 times 1 - 0 times 0 = 1 $$



                                  This is a volume in more than a 2 by 2 matrix and will be equal to 1 in all cases as the off diagonal elements remove any effect from the only value contributing to the volume as the diagonal product of 1s. It is understood in some contexts that the coordinate system is unmodified.






                                  share|cite|improve this answer











                                  $endgroup$
















                                    12












                                    12








                                    12





                                    $begingroup$

                                    There are excellent answers here that are very detailed.



                                    Here I provide a simpler answer, also discussed in wikipedia. Think of the determinant as the area (in 2D; in 3D it would be the volume, etc.) of the parallelogram made by the vectors:



                                    Parallelogram from vectors



                                    Keep in mind that the area of a parallelogram is the base $times$ height. Doing some tricks with the dot product, this yields the determinant:



                                    $$
                                    begin{vmatrix}
                                    a & b \
                                    c & d
                                    end{vmatrix}
                                    = ad - bc = Area_{parallelogram}
                                    $$



                                    You can place the unit vectors for each dimension to test the identity matrix by seeing that:
                                    $$
                                    begin{vmatrix}
                                    1 & 0 \
                                    0 & 1
                                    end{vmatrix}
                                    = ad - bc = 1 times 1 - 0 times 0 = 1 $$



                                    This is a volume in more than a 2 by 2 matrix and will be equal to 1 in all cases as the off diagonal elements remove any effect from the only value contributing to the volume as the diagonal product of 1s. It is understood in some contexts that the coordinate system is unmodified.






                                    share|cite|improve this answer











                                    $endgroup$



                                    There are excellent answers here that are very detailed.



                                    Here I provide a simpler answer, also discussed in wikipedia. Think of the determinant as the area (in 2D; in 3D it would be the volume, etc.) of the parallelogram made by the vectors:



                                    Parallelogram from vectors



                                    Keep in mind that the area of a parallelogram is the base $times$ height. Doing some tricks with the dot product, this yields the determinant:



                                    $$
                                    begin{vmatrix}
                                    a & b \
                                    c & d
                                    end{vmatrix}
                                    = ad - bc = Area_{parallelogram}
                                    $$



                                    You can place the unit vectors for each dimension to test the identity matrix by seeing that:
                                    $$
                                    begin{vmatrix}
                                    1 & 0 \
                                    0 & 1
                                    end{vmatrix}
                                    = ad - bc = 1 times 1 - 0 times 0 = 1 $$



                                    This is a volume in more than a 2 by 2 matrix and will be equal to 1 in all cases as the off diagonal elements remove any effect from the only value contributing to the volume as the diagonal product of 1s. It is understood in some contexts that the coordinate system is unmodified.







                                    share|cite|improve this answer














                                    share|cite|improve this answer



                                    share|cite|improve this answer








                                    edited Aug 19 '18 at 6:03









                                    Vass

                                    61121023




                                    61121023










                                    answered Feb 10 '17 at 2:55









                                    Mike WilliamsonMike Williamson

                                    283210




                                    283210























                                        6












                                        $begingroup$

                                        (I considered making this a comment, but I thought it might deserve more attention than a comment would receive. Upvotes and downvotes will tell if I am right or wrong).



                                        Complement about the sign of the determinant



                                        I loved the accepted answer by Jamie, but I was frustrated that it did not give more explanation about the sign of the determinant and the notion of "rotation" or "orientation" of a vector. The answer from Marc Van Leeuwen comments more on this, but maybe not enough for everyone -- at least for me -- to understand what it means for a matrix to change the orientation of the space it transforms. So I googled the issue and ended up on the following explanation which I find excellent and accessible:



                                        http://mathinsight.org/determinant_linear_transformation#lintrans3D






                                        share|cite|improve this answer











                                        $endgroup$


















                                          6












                                          $begingroup$

                                          (I considered making this a comment, but I thought it might deserve more attention than a comment would receive. Upvotes and downvotes will tell if I am right or wrong).



                                          Complement about the sign of the determinant



                                          I loved the accepted answer by Jamie, but I was frustrated that it did not give more explanation about the sign of the determinant and the notion of "rotation" or "orientation" of a vector. The answer from Marc Van Leeuwen comments more on this, but maybe not enough for everyone -- at least for me -- to understand what it means for a matrix to change the orientation of the space it transforms. So I googled the issue and ended up on the following explanation which I find excellent and accessible:



                                          http://mathinsight.org/determinant_linear_transformation#lintrans3D






                                          share|cite|improve this answer











                                          $endgroup$
















                                            6












                                            6








                                            6





                                            $begingroup$

                                            (I considered making this a comment, but I thought it might deserve more attention than a comment would receive. Upvotes and downvotes will tell if I am right or wrong).



                                            Complement about the sign of the determinant



                                            I loved the accepted answer by Jamie, but I was frustrated that it did not give more explanation about the sign of the determinant and the notion of "rotation" or "orientation" of a vector. The answer from Marc Van Leeuwen comments more on this, but maybe not enough for everyone -- at least for me -- to understand what it means for a matrix to change the orientation of the space it transforms. So I googled the issue and ended up on the following explanation which I find excellent and accessible:



                                            http://mathinsight.org/determinant_linear_transformation#lintrans3D






                                            share|cite|improve this answer











                                            $endgroup$



                                            (I considered making this a comment, but I thought it might deserve more attention than a comment would receive. Upvotes and downvotes will tell if I am right or wrong).



                                            Complement about the sign of the determinant



                                            I loved the accepted answer by Jamie, but I was frustrated that it did not give more explanation about the sign of the determinant and the notion of "rotation" or "orientation" of a vector. The answer from Marc Van Leeuwen comments more on this, but maybe not enough for everyone -- at least for me -- to understand what it means for a matrix to change the orientation of the space it transforms. So I googled the issue and ended up on the following explanation which I find excellent and accessible:



                                            http://mathinsight.org/determinant_linear_transformation#lintrans3D







                                            share|cite|improve this answer














                                            share|cite|improve this answer



                                            share|cite|improve this answer








                                            edited Nov 2 '13 at 22:37

























                                            answered Nov 2 '13 at 22:32









                                            Martin Van der LindenMartin Van der Linden

                                            1,63311027




                                            1,63311027























                                                4












                                                $begingroup$

                                                Think about a scalar equation,
                                                $$ax = b$$
                                                where we want to solve for $x$. We know we can always solve the equation if $aneq 0$, however, if $a=0$ then the answer is "it depends". If $bneq 0$, then we cannot solve it, however, if $b=0$ then there are many solutions (i.e. $x in mathbb{R}$). The key point is that the ability to solve the equation unambiguously depends on whether $a=0$.



                                                When we consider the similar equation for matrices



                                                $$mathbf{Ax} = mathbf{b}$$



                                                the question as to whether we can solve it is not so easily settled by whether $mathbf{A}=mathbf{0}$ because $mathbf{A}$ could consist of all non-zero elements and still not be solvable for $mathbf{b}neqmathbf{0}$. In fact, for two different vectors $mathbf{y}_1 neq mathbf{0}$ and $mathbf{y}_2neq mathbf{0}$ we could very well have that



                                                $$mathbf{Ay}_1 neq mathbf{0}$$
                                                and
                                                $$mathbf{Ay}_2 = mathbf{0}.$$



                                                If we think of $mathbf{y}$ as a vector, then there are some directions in which $mathbf{A}$ behaves like non-zero (this is called the row space) and other directions where $mathbf{A}$ behaves like zero (this is called the null space). The bottom line is that if $mathbf{A}$ behaves like zero in some directions, then the answer to the question "is $mathbf{Ax} = mathbf{b}$ generally solvable for any $mathbf{b}$?" is "it depends on $mathbf{b}$". More specifically, if $mathbf{b}$ is in the column space of $mathbf{A}$, then there is a solution.



                                                So is there a way that we can tell whether $mathbf{A}$ behaves like zero in some directions? Yes, it is the determinant! If $det(mathbf{A})neq 0$ then $mathbf{Ax} = mathbf{b}$ always has a solution. However if, $det(mathbf{A}) = 0$ then $mathbf{Ax} = mathbf{b}$ may or may not have a solution depending on $mathbf{b}$ and if there is one, then there are an infinite number of solutions.






                                                share|cite|improve this answer









                                                $endgroup$


















                                                  4












                                                  $begingroup$

                                                  Think about a scalar equation,
                                                  $$ax = b$$
                                                  where we want to solve for $x$. We know we can always solve the equation if $aneq 0$, however, if $a=0$ then the answer is "it depends". If $bneq 0$, then we cannot solve it, however, if $b=0$ then there are many solutions (i.e. $x in mathbb{R}$). The key point is that the ability to solve the equation unambiguously depends on whether $a=0$.



                                                  When we consider the similar equation for matrices



                                                  $$mathbf{Ax} = mathbf{b}$$



                                                  the question as to whether we can solve it is not so easily settled by whether $mathbf{A}=mathbf{0}$ because $mathbf{A}$ could consist of all non-zero elements and still not be solvable for $mathbf{b}neqmathbf{0}$. In fact, for two different vectors $mathbf{y}_1 neq mathbf{0}$ and $mathbf{y}_2neq mathbf{0}$ we could very well have that



                                                  $$mathbf{Ay}_1 neq mathbf{0}$$
                                                  and
                                                  $$mathbf{Ay}_2 = mathbf{0}.$$



                                                  If we think of $mathbf{y}$ as a vector, then there are some directions in which $mathbf{A}$ behaves like non-zero (this is called the row space) and other directions where $mathbf{A}$ behaves like zero (this is called the null space). The bottom line is that if $mathbf{A}$ behaves like zero in some directions, then the answer to the question "is $mathbf{Ax} = mathbf{b}$ generally solvable for any $mathbf{b}$?" is "it depends on $mathbf{b}$". More specifically, if $mathbf{b}$ is in the column space of $mathbf{A}$, then there is a solution.



                                                  So is there a way that we can tell whether $mathbf{A}$ behaves like zero in some directions? Yes, it is the determinant! If $det(mathbf{A})neq 0$ then $mathbf{Ax} = mathbf{b}$ always has a solution. However if, $det(mathbf{A}) = 0$ then $mathbf{Ax} = mathbf{b}$ may or may not have a solution depending on $mathbf{b}$ and if there is one, then there are an infinite number of solutions.






                                                  share|cite|improve this answer









                                                  $endgroup$
















                                                    4












                                                    4








                                                    4





                                                    $begingroup$

                                                    Think about a scalar equation,
                                                    $$ax = b$$
                                                    where we want to solve for $x$. We know we can always solve the equation if $aneq 0$, however, if $a=0$ then the answer is "it depends". If $bneq 0$, then we cannot solve it, however, if $b=0$ then there are many solutions (i.e. $x in mathbb{R}$). The key point is that the ability to solve the equation unambiguously depends on whether $a=0$.



                                                    When we consider the similar equation for matrices



                                                    $$mathbf{Ax} = mathbf{b}$$



                                                    the question as to whether we can solve it is not so easily settled by whether $mathbf{A}=mathbf{0}$ because $mathbf{A}$ could consist of all non-zero elements and still not be solvable for $mathbf{b}neqmathbf{0}$. In fact, for two different vectors $mathbf{y}_1 neq mathbf{0}$ and $mathbf{y}_2neq mathbf{0}$ we could very well have that



                                                    $$mathbf{Ay}_1 neq mathbf{0}$$
                                                    and
                                                    $$mathbf{Ay}_2 = mathbf{0}.$$



                                                    If we think of $mathbf{y}$ as a vector, then there are some directions in which $mathbf{A}$ behaves like non-zero (this is called the row space) and other directions where $mathbf{A}$ behaves like zero (this is called the null space). The bottom line is that if $mathbf{A}$ behaves like zero in some directions, then the answer to the question "is $mathbf{Ax} = mathbf{b}$ generally solvable for any $mathbf{b}$?" is "it depends on $mathbf{b}$". More specifically, if $mathbf{b}$ is in the column space of $mathbf{A}$, then there is a solution.



                                                    So is there a way that we can tell whether $mathbf{A}$ behaves like zero in some directions? Yes, it is the determinant! If $det(mathbf{A})neq 0$ then $mathbf{Ax} = mathbf{b}$ always has a solution. However if, $det(mathbf{A}) = 0$ then $mathbf{Ax} = mathbf{b}$ may or may not have a solution depending on $mathbf{b}$ and if there is one, then there are an infinite number of solutions.






                                                    share|cite|improve this answer









                                                    $endgroup$



                                                    Think about a scalar equation,
                                                    $$ax = b$$
                                                    where we want to solve for $x$. We know we can always solve the equation if $aneq 0$, however, if $a=0$ then the answer is "it depends". If $bneq 0$, then we cannot solve it, however, if $b=0$ then there are many solutions (i.e. $x in mathbb{R}$). The key point is that the ability to solve the equation unambiguously depends on whether $a=0$.



                                                    When we consider the similar equation for matrices



                                                    $$mathbf{Ax} = mathbf{b}$$



                                                    the question as to whether we can solve it is not so easily settled by whether $mathbf{A}=mathbf{0}$ because $mathbf{A}$ could consist of all non-zero elements and still not be solvable for $mathbf{b}neqmathbf{0}$. In fact, for two different vectors $mathbf{y}_1 neq mathbf{0}$ and $mathbf{y}_2neq mathbf{0}$ we could very well have that



                                                    $$mathbf{Ay}_1 neq mathbf{0}$$
                                                    and
                                                    $$mathbf{Ay}_2 = mathbf{0}.$$



                                                    If we think of $mathbf{y}$ as a vector, then there are some directions in which $mathbf{A}$ behaves like non-zero (this is called the row space) and other directions where $mathbf{A}$ behaves like zero (this is called the null space). The bottom line is that if $mathbf{A}$ behaves like zero in some directions, then the answer to the question "is $mathbf{Ax} = mathbf{b}$ generally solvable for any $mathbf{b}$?" is "it depends on $mathbf{b}$". More specifically, if $mathbf{b}$ is in the column space of $mathbf{A}$, then there is a solution.



                                                    So is there a way that we can tell whether $mathbf{A}$ behaves like zero in some directions? Yes, it is the determinant! If $det(mathbf{A})neq 0$ then $mathbf{Ax} = mathbf{b}$ always has a solution. However if, $det(mathbf{A}) = 0$ then $mathbf{Ax} = mathbf{b}$ may or may not have a solution depending on $mathbf{b}$ and if there is one, then there are an infinite number of solutions.







                                                    share|cite|improve this answer












                                                    share|cite|improve this answer



                                                    share|cite|improve this answer










                                                    answered May 17 '12 at 3:18









                                                    TpofofnTpofofn

                                                    3,6271427




                                                    3,6271427























                                                        2












                                                        $begingroup$

                                                        One way to treat define the determinant which makes clear the relation between all the various notions you mentioned is as follows:



                                                        Given a vector space $E$ of dimmension $n$ over the field $K$ and a basis $B=(b_1,...,b_n)$ of $E$, the determinant is the unique (nonzero) alternating multilinear $n$-form $phi$ of $E$ which satisfies $phi(b_1,...,b_n)=1$.



                                                        This simply means that the determinant is a function $phi$ which takes a tuple $(x_1,...,x_n)$ of $n$ vectors of $E$ and returns a scalar from the field $K$, such that



                                                        (1) $phi$ is linear in each of the $n$ variables $(x_1,...,x_n)$ (it's "multilinear")



                                                        (2) if two of the $x_i$'s are equal, then $phi(x_1,...,x_n)=0$ ($phi$ is "alternating")



                                                        (3) It turns out that the set of functions $phi$ satisfying the two above properties are all multiples of eachother. So we choose a basis $B$ of $e$ and say that the determinant is the function $phi$ satisfying the above properties which maps $B$ to $1$.



                                                        Of course it's not immediately obvious that such a function $phi$ exists and is unique!



                                                        To simplify slightly we will take the vector space $E$ to be $K^n$ and the basis $B$ to be the canonical basis.



                                                        It turns out that the determinant satisfies the miraculous property that $det(x_1,...,x_n) neq 0$ if and only if $(x_1,...,x_n)$ is a basis.



                                                        Now... given $n$ vectors $x_1,...,x_n$ such that for the coordinates in the basis $B$ of $x_i$ are $(a_{i,1},...,a_{i,n})$, the determinant of the $n$-vectors $x_1,...,x_n$ can be shown to be equal to



                                                        $sum_{sigma in S_n} sgn(sigma)a_{1,sigma(1)}...a_{n,sigma(n)}$



                                                        which should be familiar to you as the expression for the determinant in terms of permutations. Here $S_n$ is the symmetric group, i.e the set of permutations of ${1,2,..,n}$ and $sgn(sigma)$ is the signature of the permutation $sigma$.



                                                        To make the link between the determinant of a set of $n$ vectors to the determinant of a matrix, just note that the matrix $A=(a_{i,j})$ is exactly them matrix whose column vectors are $x_1,...,x_n$.



                                                        Thus when we take the determinant of a matrix, what we are really doing is evaluating a function in terms of the $n$ column vectors. We said earlier that this function is nonzero if and only if the $n$ vectors form a basis - in other words, if and only if the matrix is of full rank, i.e iff its invertible.



                                                        So the abstract definition the determinant as a function which maps a set of vectors to the scalar field (while obeying some nice properties like linearity) is equivalent to a function from matrices to the scalar field which is nonzero exactly when the matrix is invertible. Moreover, this function turns out to be multiplicative! (Consequently, the restriction of this function to the set of invertible matrices gives is a group homomorphism from $(Gl_n(K), times)$ to $(K/{0},*)$.



                                                        The expression of the determinant of a matrix in terms of permutations can be used to derive many of the nice properties you are familiar with, for example




                                                        • a matrix and its transpose have the same det


                                                        • det of a triangular matrix is the product of the diagonal elements


                                                        • the Laplace-formula a.k.a cofactor expansion which tells you how to calculate the determinant in terms of a weighted sum of determinants of submatrices:



                                                        $det(A)=sum_{i=1}^{n}(-1)^{i+j}a_{i,j}Delta_{i,j}$



                                                        where $Delta_{i,j}$ is the determinant of the matrix obtained from $A$ by removing the row $i$ and the column $j$, known as the minor $(i,j)$.






                                                        share|cite|improve this answer











                                                        $endgroup$


















                                                          2












                                                          $begingroup$

                                                          One way to treat define the determinant which makes clear the relation between all the various notions you mentioned is as follows:



                                                          Given a vector space $E$ of dimmension $n$ over the field $K$ and a basis $B=(b_1,...,b_n)$ of $E$, the determinant is the unique (nonzero) alternating multilinear $n$-form $phi$ of $E$ which satisfies $phi(b_1,...,b_n)=1$.



                                                          This simply means that the determinant is a function $phi$ which takes a tuple $(x_1,...,x_n)$ of $n$ vectors of $E$ and returns a scalar from the field $K$, such that



                                                          (1) $phi$ is linear in each of the $n$ variables $(x_1,...,x_n)$ (it's "multilinear")



                                                          (2) if two of the $x_i$'s are equal, then $phi(x_1,...,x_n)=0$ ($phi$ is "alternating")



                                                          (3) It turns out that the set of functions $phi$ satisfying the two above properties are all multiples of eachother. So we choose a basis $B$ of $e$ and say that the determinant is the function $phi$ satisfying the above properties which maps $B$ to $1$.



                                                          Of course it's not immediately obvious that such a function $phi$ exists and is unique!



                                                          To simplify slightly we will take the vector space $E$ to be $K^n$ and the basis $B$ to be the canonical basis.



                                                          It turns out that the determinant satisfies the miraculous property that $det(x_1,...,x_n) neq 0$ if and only if $(x_1,...,x_n)$ is a basis.



                                                          Now... given $n$ vectors $x_1,...,x_n$ such that for the coordinates in the basis $B$ of $x_i$ are $(a_{i,1},...,a_{i,n})$, the determinant of the $n$-vectors $x_1,...,x_n$ can be shown to be equal to



                                                          $sum_{sigma in S_n} sgn(sigma)a_{1,sigma(1)}...a_{n,sigma(n)}$



                                                          which should be familiar to you as the expression for the determinant in terms of permutations. Here $S_n$ is the symmetric group, i.e the set of permutations of ${1,2,..,n}$ and $sgn(sigma)$ is the signature of the permutation $sigma$.



                                                          To make the link between the determinant of a set of $n$ vectors to the determinant of a matrix, just note that the matrix $A=(a_{i,j})$ is exactly them matrix whose column vectors are $x_1,...,x_n$.



                                                          Thus when we take the determinant of a matrix, what we are really doing is evaluating a function in terms of the $n$ column vectors. We said earlier that this function is nonzero if and only if the $n$ vectors form a basis - in other words, if and only if the matrix is of full rank, i.e iff its invertible.



                                                          So the abstract definition the determinant as a function which maps a set of vectors to the scalar field (while obeying some nice properties like linearity) is equivalent to a function from matrices to the scalar field which is nonzero exactly when the matrix is invertible. Moreover, this function turns out to be multiplicative! (Consequently, the restriction of this function to the set of invertible matrices gives is a group homomorphism from $(Gl_n(K), times)$ to $(K/{0},*)$.



                                                          The expression of the determinant of a matrix in terms of permutations can be used to derive many of the nice properties you are familiar with, for example




                                                          • a matrix and its transpose have the same det


                                                          • det of a triangular matrix is the product of the diagonal elements


                                                          • the Laplace-formula a.k.a cofactor expansion which tells you how to calculate the determinant in terms of a weighted sum of determinants of submatrices:



                                                          $det(A)=sum_{i=1}^{n}(-1)^{i+j}a_{i,j}Delta_{i,j}$



                                                          where $Delta_{i,j}$ is the determinant of the matrix obtained from $A$ by removing the row $i$ and the column $j$, known as the minor $(i,j)$.






                                                          share|cite|improve this answer











                                                          $endgroup$
















                                                            2












                                                            2








                                                            2





                                                            $begingroup$

                                                            One way to treat define the determinant which makes clear the relation between all the various notions you mentioned is as follows:



                                                            Given a vector space $E$ of dimmension $n$ over the field $K$ and a basis $B=(b_1,...,b_n)$ of $E$, the determinant is the unique (nonzero) alternating multilinear $n$-form $phi$ of $E$ which satisfies $phi(b_1,...,b_n)=1$.



                                                            This simply means that the determinant is a function $phi$ which takes a tuple $(x_1,...,x_n)$ of $n$ vectors of $E$ and returns a scalar from the field $K$, such that



                                                            (1) $phi$ is linear in each of the $n$ variables $(x_1,...,x_n)$ (it's "multilinear")



                                                            (2) if two of the $x_i$'s are equal, then $phi(x_1,...,x_n)=0$ ($phi$ is "alternating")



                                                            (3) It turns out that the set of functions $phi$ satisfying the two above properties are all multiples of eachother. So we choose a basis $B$ of $e$ and say that the determinant is the function $phi$ satisfying the above properties which maps $B$ to $1$.



                                                            Of course it's not immediately obvious that such a function $phi$ exists and is unique!



                                                            To simplify slightly we will take the vector space $E$ to be $K^n$ and the basis $B$ to be the canonical basis.



                                                            It turns out that the determinant satisfies the miraculous property that $det(x_1,...,x_n) neq 0$ if and only if $(x_1,...,x_n)$ is a basis.



                                                            Now... given $n$ vectors $x_1,...,x_n$ such that for the coordinates in the basis $B$ of $x_i$ are $(a_{i,1},...,a_{i,n})$, the determinant of the $n$-vectors $x_1,...,x_n$ can be shown to be equal to



                                                            $sum_{sigma in S_n} sgn(sigma)a_{1,sigma(1)}...a_{n,sigma(n)}$



                                                            which should be familiar to you as the expression for the determinant in terms of permutations. Here $S_n$ is the symmetric group, i.e the set of permutations of ${1,2,..,n}$ and $sgn(sigma)$ is the signature of the permutation $sigma$.



                                                            To make the link between the determinant of a set of $n$ vectors to the determinant of a matrix, just note that the matrix $A=(a_{i,j})$ is exactly them matrix whose column vectors are $x_1,...,x_n$.



                                                            Thus when we take the determinant of a matrix, what we are really doing is evaluating a function in terms of the $n$ column vectors. We said earlier that this function is nonzero if and only if the $n$ vectors form a basis - in other words, if and only if the matrix is of full rank, i.e iff its invertible.



                                                            So the abstract definition the determinant as a function which maps a set of vectors to the scalar field (while obeying some nice properties like linearity) is equivalent to a function from matrices to the scalar field which is nonzero exactly when the matrix is invertible. Moreover, this function turns out to be multiplicative! (Consequently, the restriction of this function to the set of invertible matrices gives is a group homomorphism from $(Gl_n(K), times)$ to $(K/{0},*)$.



                                                            The expression of the determinant of a matrix in terms of permutations can be used to derive many of the nice properties you are familiar with, for example




                                                            • a matrix and its transpose have the same det


                                                            • det of a triangular matrix is the product of the diagonal elements


                                                            • the Laplace-formula a.k.a cofactor expansion which tells you how to calculate the determinant in terms of a weighted sum of determinants of submatrices:



                                                            $det(A)=sum_{i=1}^{n}(-1)^{i+j}a_{i,j}Delta_{i,j}$



                                                            where $Delta_{i,j}$ is the determinant of the matrix obtained from $A$ by removing the row $i$ and the column $j$, known as the minor $(i,j)$.






                                                            share|cite|improve this answer











                                                            $endgroup$



                                                            One way to treat define the determinant which makes clear the relation between all the various notions you mentioned is as follows:



                                                            Given a vector space $E$ of dimmension $n$ over the field $K$ and a basis $B=(b_1,...,b_n)$ of $E$, the determinant is the unique (nonzero) alternating multilinear $n$-form $phi$ of $E$ which satisfies $phi(b_1,...,b_n)=1$.



                                                            This simply means that the determinant is a function $phi$ which takes a tuple $(x_1,...,x_n)$ of $n$ vectors of $E$ and returns a scalar from the field $K$, such that



                                                            (1) $phi$ is linear in each of the $n$ variables $(x_1,...,x_n)$ (it's "multilinear")



                                                            (2) if two of the $x_i$'s are equal, then $phi(x_1,...,x_n)=0$ ($phi$ is "alternating")



                                                            (3) It turns out that the set of functions $phi$ satisfying the two above properties are all multiples of eachother. So we choose a basis $B$ of $e$ and say that the determinant is the function $phi$ satisfying the above properties which maps $B$ to $1$.



                                                            Of course it's not immediately obvious that such a function $phi$ exists and is unique!



                                                            To simplify slightly we will take the vector space $E$ to be $K^n$ and the basis $B$ to be the canonical basis.



                                                            It turns out that the determinant satisfies the miraculous property that $det(x_1,...,x_n) neq 0$ if and only if $(x_1,...,x_n)$ is a basis.



                                                            Now... given $n$ vectors $x_1,...,x_n$ such that for the coordinates in the basis $B$ of $x_i$ are $(a_{i,1},...,a_{i,n})$, the determinant of the $n$-vectors $x_1,...,x_n$ can be shown to be equal to



                                                            $sum_{sigma in S_n} sgn(sigma)a_{1,sigma(1)}...a_{n,sigma(n)}$



                                                            which should be familiar to you as the expression for the determinant in terms of permutations. Here $S_n$ is the symmetric group, i.e the set of permutations of ${1,2,..,n}$ and $sgn(sigma)$ is the signature of the permutation $sigma$.



                                                            To make the link between the determinant of a set of $n$ vectors to the determinant of a matrix, just note that the matrix $A=(a_{i,j})$ is exactly them matrix whose column vectors are $x_1,...,x_n$.



                                                            Thus when we take the determinant of a matrix, what we are really doing is evaluating a function in terms of the $n$ column vectors. We said earlier that this function is nonzero if and only if the $n$ vectors form a basis - in other words, if and only if the matrix is of full rank, i.e iff its invertible.



                                                            So the abstract definition the determinant as a function which maps a set of vectors to the scalar field (while obeying some nice properties like linearity) is equivalent to a function from matrices to the scalar field which is nonzero exactly when the matrix is invertible. Moreover, this function turns out to be multiplicative! (Consequently, the restriction of this function to the set of invertible matrices gives is a group homomorphism from $(Gl_n(K), times)$ to $(K/{0},*)$.



                                                            The expression of the determinant of a matrix in terms of permutations can be used to derive many of the nice properties you are familiar with, for example




                                                            • a matrix and its transpose have the same det


                                                            • det of a triangular matrix is the product of the diagonal elements


                                                            • the Laplace-formula a.k.a cofactor expansion which tells you how to calculate the determinant in terms of a weighted sum of determinants of submatrices:



                                                            $det(A)=sum_{i=1}^{n}(-1)^{i+j}a_{i,j}Delta_{i,j}$



                                                            where $Delta_{i,j}$ is the determinant of the matrix obtained from $A$ by removing the row $i$ and the column $j$, known as the minor $(i,j)$.







                                                            share|cite|improve this answer














                                                            share|cite|improve this answer



                                                            share|cite|improve this answer








                                                            edited Apr 24 '17 at 16:24

























                                                            answered Apr 24 '17 at 15:35









                                                            Joshua BenabouJoshua Benabou

                                                            2,557626




                                                            2,557626























                                                                0












                                                                $begingroup$

                                                                Imagine a completely general system of equations



                                                                $$a_{11} x_1 + a_{12} x_2 + a_{13} x_3 = b_1$$
                                                                $$a_{21} x_1 + a_{22} x_2 + a_{23} x_3 = b_2$$
                                                                $$a_{31} x_1 + a_{32} x_2 + a_{33} x_3 = b_3$$



                                                                If we solve for the variables $x_i$ in terms of the other variables and write the results in lowest terms, we'll see that the expressions for each $x_i$ all have the same functions of $a_{ij}$ in the denominator. (Say that we work over the integers.) This expression is (up to a unit) the determinant of the system.



                                                                If you pick some systematic way of solving $n times n$ systems, say Gaussian elimination, you can use it to crank out a formula for this determinant.



                                                                I think this is a lot more natural than the other approaches because you start with something straightforward and common like a system of linear equations, then you put your head down and solve it, and out pops this notion.



                                                                Of course this only gives you the answer up to a sign, but this actually makes sense, because there's an arbitrary choice of sign going in.



                                                                Garibaldi has a paper that presents this approach and some related ones, entitled The determinant and characteristic polynomial are not ad hoc constructions. (To formalize this you want to bring in a little ring theory so that you have formal notions of indeterminates and so forth.)






                                                                share|cite|improve this answer











                                                                $endgroup$


















                                                                  0












                                                                  $begingroup$

                                                                  Imagine a completely general system of equations



                                                                  $$a_{11} x_1 + a_{12} x_2 + a_{13} x_3 = b_1$$
                                                                  $$a_{21} x_1 + a_{22} x_2 + a_{23} x_3 = b_2$$
                                                                  $$a_{31} x_1 + a_{32} x_2 + a_{33} x_3 = b_3$$



                                                                  If we solve for the variables $x_i$ in terms of the other variables and write the results in lowest terms, we'll see that the expressions for each $x_i$ all have the same functions of $a_{ij}$ in the denominator. (Say that we work over the integers.) This expression is (up to a unit) the determinant of the system.



                                                                  If you pick some systematic way of solving $n times n$ systems, say Gaussian elimination, you can use it to crank out a formula for this determinant.



                                                                  I think this is a lot more natural than the other approaches because you start with something straightforward and common like a system of linear equations, then you put your head down and solve it, and out pops this notion.



                                                                  Of course this only gives you the answer up to a sign, but this actually makes sense, because there's an arbitrary choice of sign going in.



                                                                  Garibaldi has a paper that presents this approach and some related ones, entitled The determinant and characteristic polynomial are not ad hoc constructions. (To formalize this you want to bring in a little ring theory so that you have formal notions of indeterminates and so forth.)






                                                                  share|cite|improve this answer











                                                                  $endgroup$
















                                                                    0












                                                                    0








                                                                    0





                                                                    $begingroup$

                                                                    Imagine a completely general system of equations



                                                                    $$a_{11} x_1 + a_{12} x_2 + a_{13} x_3 = b_1$$
                                                                    $$a_{21} x_1 + a_{22} x_2 + a_{23} x_3 = b_2$$
                                                                    $$a_{31} x_1 + a_{32} x_2 + a_{33} x_3 = b_3$$



                                                                    If we solve for the variables $x_i$ in terms of the other variables and write the results in lowest terms, we'll see that the expressions for each $x_i$ all have the same functions of $a_{ij}$ in the denominator. (Say that we work over the integers.) This expression is (up to a unit) the determinant of the system.



                                                                    If you pick some systematic way of solving $n times n$ systems, say Gaussian elimination, you can use it to crank out a formula for this determinant.



                                                                    I think this is a lot more natural than the other approaches because you start with something straightforward and common like a system of linear equations, then you put your head down and solve it, and out pops this notion.



                                                                    Of course this only gives you the answer up to a sign, but this actually makes sense, because there's an arbitrary choice of sign going in.



                                                                    Garibaldi has a paper that presents this approach and some related ones, entitled The determinant and characteristic polynomial are not ad hoc constructions. (To formalize this you want to bring in a little ring theory so that you have formal notions of indeterminates and so forth.)






                                                                    share|cite|improve this answer











                                                                    $endgroup$



                                                                    Imagine a completely general system of equations



                                                                    $$a_{11} x_1 + a_{12} x_2 + a_{13} x_3 = b_1$$
                                                                    $$a_{21} x_1 + a_{22} x_2 + a_{23} x_3 = b_2$$
                                                                    $$a_{31} x_1 + a_{32} x_2 + a_{33} x_3 = b_3$$



                                                                    If we solve for the variables $x_i$ in terms of the other variables and write the results in lowest terms, we'll see that the expressions for each $x_i$ all have the same functions of $a_{ij}$ in the denominator. (Say that we work over the integers.) This expression is (up to a unit) the determinant of the system.



                                                                    If you pick some systematic way of solving $n times n$ systems, say Gaussian elimination, you can use it to crank out a formula for this determinant.



                                                                    I think this is a lot more natural than the other approaches because you start with something straightforward and common like a system of linear equations, then you put your head down and solve it, and out pops this notion.



                                                                    Of course this only gives you the answer up to a sign, but this actually makes sense, because there's an arbitrary choice of sign going in.



                                                                    Garibaldi has a paper that presents this approach and some related ones, entitled The determinant and characteristic polynomial are not ad hoc constructions. (To formalize this you want to bring in a little ring theory so that you have formal notions of indeterminates and so forth.)







                                                                    share|cite|improve this answer














                                                                    share|cite|improve this answer



                                                                    share|cite|improve this answer








                                                                    edited Mar 3 at 21:44

























                                                                    answered Mar 1 at 5:53









                                                                    Daniel McLauryDaniel McLaury

                                                                    15.9k32981




                                                                    15.9k32981

















                                                                        protected by Community Apr 20 '15 at 22:34



                                                                        Thank you for your interest in this question.
                                                                        Because it has attracted low-quality or spam answers that had to be removed, posting an answer now requires 10 reputation on this site (the association bonus does not count).



                                                                        Would you like to answer one of these unanswered questions instead?



                                                                        Popular posts from this blog

                                                                        Wiesbaden

                                                                        To store a contact into the json file from server.js file using a class in NodeJS

                                                                        Marschland